Xradiologist

Written examination Includes-2012 OSU, Chicago, Columbus Ohio2011 OSU, Chicago, Columbus Ohio, GCH and POH Michigan, Texas2010 OSU, Chicago, Columbus Ohio, GCH Michigan2009 OSU, Chicago, Columbus Ohio, GCH Michigan2008 OSU, Chicago, Columbus Ohio, GCH MichiganOn the exam, most questions ask- which is false, or, which is true. However, because this is a combination of multiple programs and multiple years, some have more than one correct answer. For example there may be multiple true and multiple false answers on a particular topic.Genitourinary Section1. Bladder diverticuliMost common on bladder dome- FalseResult in stasis/stones- TrueMost common posterior lateral- TrueDiscussion- The most common locations are the lateral walls (62%) > posterior wall (21%)> ureteral orifice > dome. Narrow neck diverticuli cause stasis/stones.>90% are in men because of prostate obstruction. 2. Bosniak classification of a 2.5 cm renal mass with Hounsfield units of 80 and does not enhance (may have also said slightly thickened calcification)?Category 2Discussion- Category 1- A benign simple cyst with a hairline-thin wall that does not contain septae, calcifications, or solid components. It measures water density, and does not enhance with contrast material.Category 2- A benign cyst that may contain a few hairline-thin septae. Fine calcification or a short segment of slightly thickened calcification may be present in the wall or septae. Uniformly high-attenuation lesions (< 3 cm) that are sharply marginated and do not enhance are included in this group.Category 2F- These well marginated cysts may contain an increased number of hairline-thin septae, with possible minimal perceived enhancement or thickening of the septae or wall.? The cyst may contain calcification that may be thick and nodular, but there are no enhancing soft-tissue components. Totally intrarenal nonenhancing high-attenuation renal lesions that are 3 cm or larger are also included in this category. ?Category 3- These lesions are indeterminate cystic masses that have thickened, irregular walls or septae in which enhancement can be seen.Category 4- These lesions are clearly malignant cystic masses that not only have all the characteristics of category 3 lesions, but also contain enhancing soft-tissue components adjacent to but independent of the wall or septae.3. Which are true statements about Xanthogranulomatous pyelonephritis?Low T1 and T2 signal- FalseT1 iso or hyperintense- lipid laden macrophages- TrueT2 iso to slightly hypointense- TrueCalcifications- TrueObstruction- TrueAbsent nephrogram- TrueDiscussion- High T1 and low T2.Total or focally absent nephrogram in 80%Proteums mirabilis, E. coli, S. aureus.Increased risk of carcinoma.Staghorn calculus in 75%.4. True of renal cell carcinomaDirect correlation of size to incidence of metastasis- TrueSmall rcc can look like angiomyolipoma- TrueDiscussion-Fat poor AML is impossible to distinguish from RCC5. Associated with Von Hippel Lindau diseasePancreatic cysts- TrueAML’s- FalsePheochromocytoma- TrueRCC- TrueHemangioblastoma- TrueCafe au lait spots- FalseLymphangiomyomatosis- FalseLymphatic spread of tumor- TrueBrain aneurysms- TrueDiscussion-Although AML’s can occur with Von Hippel Lindau, it is rare and should be false for test purposes. AML’s are associated with tuberous sclerosis.Cafe au lait spots with NF-1.Lymphangiomyomatosis is with tuberous sclerosis.Hemangioblastomas can get aneurysmsRCC can spread via lymphaticsTrue of urethral injuryAnterior urethral injury most common iatrogenic cause- FalseGrade 2 shows extravasation into the perineum- FalseGrade 2 injuries involve the urethra above and below the diaphragm- FalseNeed to catheterize bladder before performing urethrogram- FalseGrade 2 is usually posterior injury- TruePosterior injury is most common secondary to pelvic fractures- TrueDiscussion-Posterior urethra (Prostatic urethra - Membranous urethra)Anterior urethra (Bulbar urethra - Penile urethra)Most common iatrogenic cause is straddle injury to perineum which damages the bulbar urethra.Grade 1 Posterior urethra intact but stretchedGrade 2 is only above urogenital diaphragm and will show extravasation into the extraperitoneal pelvis.Grade 3 is above and below and will show extravasation into the perineum.7. Testicular torsion80% salvageable at 12 hours- FalseNegative ultrasound does not obviate need for nuclear scan- TrueEnlarged epididymis- TrueLack of arterial flow- TrueHypoechoic early finding- TrueHeterogeneous echogenicity- TrueHemorrhage seen in testicle or tunica- FalseThickened scrotum and hydrocele- TrueDiscussion-At 12 hours only 20% are salvageableStarts as hypoechoic and converts to heterogeneous between 24 h and 10 d.8. Enlarged kidneyRenal vein thrombus- TrueTuberculosis- FalseRenal artery occlusion- FalseDiscussion-Genitourinary tract is the second most common site of TB behind pulmonary.It is associated with autonephrectomy and a small shrunken calcified kidney or putty kidney (not enlarged). Retroperitoneal fibrosisMedial deviation of the ureters.Lateral deviation of the upper ureters.Lower pole renal mass- FalseUreter that traverses behind the IVC- FalseHorseshoe kidney with malrotation- TrueDiscussion-Lateral deviation of upper ureters-Lymphadenopathy, aortic aneurysm, retroperitoneal hematoma.Lateral deviation of the lower ureters-Pelvic mass like uterine fibroid.Lower pole renal mass and retrocaval ureter would both cause medial deviation.Renal vein thrombosis-More common in neonates than adults- TrueMost common cause in adults is dehydration- FalseMost common cause in adults is nephritic syndrome- TrueR>L- FalseCan look like chronic renal artery stenosis- MaybeDiscussion-Dehydration and sepsis are the most common causes in kids.Bilateral more common in kids.L>R because L renal vein is longer.Doppler can show high resistance waveform with diastolic flow reversal.Causes of medullary nephrocalcinosisHyperparathyroid- TrueRenal tubular acidosis type 1- TrueMedullary sponge kidney- TrueDiscussion-40%, 20%, 20% respectivelyMedullary is 95% of nephrocalcinosis, Cortical is 5%Causes of medullary nephrocalcinosisRenal Tubular AcidosisMedullary sponge kidneyChronic pyeloPapillary necrosisCushingsHyperparathyroidHyper/hypothyroidIdiopahtic HypercalcemiaRenal TBHypercalciuriaSickle CellSarcoidVit D excessMilk AlkaliMalignancyAmphotericin BCauses of cortical nephrocalcinosisChronic glomerulonephritis- TrueAcute cortical necrosis- TrueRejected renal transplant- TrueAlport’s syndrome- TrueDiscussion-Medullary is 95% of nephrocalcinosis, Cortical is 5%Causes of cortical nephrocalcinosis-Chronic GlomerulonephritisAcute Cortical necrosis—pregnancy, shock, infectionAlports—glomerulonephritis and hearing loss,x-linkedAIDS nephropathyOxalosisChronic hypercalcemiaEthylene GlycolSickle Cell Rejected renal transplantMedullary sponge kidneyMedullary nephrocalcinosis- TrueIncreased echogenicity of pyramids- TrueElevated AFP in 17th week of gestation, previous cesarean section, placenta covering cervical os/ previa.Placenta percretaDiscussion-Spectrum of Placenta accreta involves 3 variantsPlacenta accreta vera (75-80%)Placenta attached to myometrium without invasion.Placenta increta (15%)Chorionic villi invade the myometrium.Placenta percreta (5%)Penetration of chorionic villi through serosaMay invade bladder, rectum and parametriumIrregular bladder wall on USAdenomyosisCalled endometriosis externa- FalseNeed junctional zone thickness greater than 5mm to diagnose- FalseNeed junctional zone thickness greater than 12mm to diagnose- TrueDiscussion-In older literature endometriosis was subclassified-Endometriosis interna- Now called adenomyosisEndometriosis externa- Now just called endometriosisRegarding peripelvic cystsCommonly obstruct causing hydronephrosis- FalseCan look like hydronephrosis on ultrasound- TrueLymphatic etiology- TrueInterdigitates between calyces and infundibula- TrueDiscussion-Peripelvic cystsLymphatic etiology.Usually multiple, small and bilateral.Stretches calyces and infundibula.Mimics hydronephrosis on US.Parapelvic cystsIndents renal sinusDisplaces collecting system.Usually solitary.Can compress vessels and collecting system (which can cause hydro).Which is false regarding acute tubular necrosis in a transplanted kidney?Decreased perfusion and decreased excretion- FalseDiscussion-Will have normal perfusion with decreased excretionOther findings with transplant rejection-RI > 0.9Decreased cortical echogenicityDiminished corticomedullary differentiation.Renal enlargement (acute rejection)Renal atrophy (chronic rejection)Soft tissue rim sign?Edema surrounding a stone stuck in a ureterMyocardial PET scanNo answers recalledDiscussion-Mismatched defect- Decreased perfusion but enhanced metabolism of fatty acids indicates viable myocardium.Matched defect- Perfusion and metabolism of fatty acids decreased indicates non-viable myocardium.Free fatty acids are the predominant metabolic substrateGlucose utilization- insulin levels increase after carbohydrate intake.Fasting for 4 hours switches to predominantly fatty acid metabolism.Autosomal dominant polycystic kidney disease, which is true?>2 cysts in each of the bilateral kidneys by age 30- Probably correctMales > Females- FalseAutosomal dominant with low penetrance- FalseCommonly associated with oligohydramnios and hydronephrosis- FalsePresents in infancy- FalseDiscussion-Male=Female 1:1>5 cysts 18-29 years old>6 cysts 30-44 years old>6 cysts in females and >9 in males 45-59 years old.Almost 100% penetrancePresents in middle agesRenal TBBegins in calyces- FalseInvolvement of bladder can cause reflux- TrueCT is better for detecting pyeloinfundibular stricture and papillary necrosis- MaybeCalcification of the bladder is common- False50% have concomitant lung findings- FalseSeen in 20% of patients with pulmonary TB- FalseOccurs in the urinary bladder first- FalseDiscussion-Begins in the cortex in peri glomerular capillaries and forms coalescing cortical granulomas.In patients with renal TB, <50% have pulmonary TB, 5% have active cavitary TB.Occurs in the kidney first then bladder.IVP is better for detecting pyeloinfundibular stricture and papillary necrosis but is rarely used now. If the question specifically compares the two it is false. If it says that CT is good for detecting pyeloinfundibular stricture and papillary necrosis, it is true.Calcification of bladder is rare.Keyhole signPosterior urethral valvesDiscussion-Congenital and only in male patients.3 types of abnormal membranes/ valves in posterior urethraI- above veramontanumII- @ veramontanum, normal variant, doesn’t obstructIII- Diaphragm like membrane below veramontanum24. Etiology of a primary Ureteropelvic junction obstructionAdynamic/aperistaltic segment of upper ureter- TrueNot caused by reflux or obstruction- TrueCan be caused by a crossing vessel- TrueMost commonly caused by a crossing vessel- FalseObstruction of ureterovesicular junction- FalseMechanical obstruction- Maybe (if this includes a crossing vessel)Discussion-Primary UPJ obstructionsIntrinsic cause-Primarily a functional obstruction due to adynamic segment. (Most common cause of Primary UPJ obstruction)Extrinsic causes-Aberrant vessels to lower pole usually anterior to UPJ (95% of extrinsic obstructions but only 25-39% of all Primary UPJ obstructions).Kinks, Adventitial bands, Renal cysts, Aortic aneurysm.Secondary UPJ obstructionsStones, XGP, Infection, Ischemia, trauma.Obstruction of ureterovesicular junction is false because that occurs with primary megaureter.25. Placenta within 2cm of internal os without covering it.Marginal placenta- FalseDiscussion-The question describes a low lying placenta.Grade 1- Low lying placenta- within 0.5- 5 cm of internal os.Grade 2- Marginal placenta- reaches margin of internal os but does not cover.Grade 3- Partial previa- partially covers internal os.Grade 4- Complete previa- completely covers internal os.26. Cushing’s syndrome associated with Low density adrenal nodule- FalseThickened bilateral adrenal glands- True27. VaricoceleR > L- FalseAV malformation of the pampiniform plexus- FalseInvolves dilation of the creamasteric plexus- True?Diagnosed when veins are < 3 mm- FalseAccentuated in the supine position- FalseDiscussion-It is dilation of the pampiniform plexus, not an AV malformation. The creamasteric vein is part of the pampiniform plexus.98% are on the LeftNot AV malformationDiagnosed when veins are > 2mm.Accentuated in upright position.28. Cryptorchidism5 times increased risk of cancer- FalseRisk of cancer in contralateral testicle- TrueAssociated with agenesis of Vas Deferens- TrueAssociated with agenesis of the epididymis- TrueIncreased risk of torsion on the affected side- TrueTesticle is usually stuck in abdomen- FalseDiscussion-30- 50 x increased risk of cancerTesticle usually in the inguinal canal29. Elevated LH/FSH ratio in pre-menopausal female with several months of amenorrhea is most likely due to?PCOD- TrueAdenomyomatosisEndometrial cancerEndometriosis30. Renal infarct on CT (alternate question said renal trauma)Absent nephrogramReversal of flow from the IVC into the renal reinSubcortical rim signAll of the above- TrueDiscussion-Can have a focally or completely absent nephrogram.Subcortical rim sign helps differentiate between pyelo and infarct.In pyelo the entire wedge (medulla to cortex) is hypoattenuating.With infarct, there will be a small subcortical rim of normal enhancement because this is supplied by a perforating renal capsular artery which is an early branch from the renal artery.This sign is only seen in about 50% of infarcts.31. Page kidneySubcapsular compressionDiscussion-Hypertension caused by activation of Renin Angiotensin system because of compression of the renal vessels by a subcapsular fluid collection (hematoma, seroma, urinoma).32. Patient with ureteral injury 2 years ago now has a septated perinephric lesion causing obstruction.Lymphocele- CorrectUrinomaAbscessHematoma33. Benign hypoechoic well defined intratesticular massEpidermoid- TrueSpermatocele- FalseChoriocarcinoma- FalseSeminoma- FalseDiscussion-Epidermoid has onion skin appearance on ultrasoundChoriocarcinoma and Seminoma are malignant34. Adrenal neoplasms?Most common are Mets and adenoma- TrueAdenomas are hyperdense on noncontrast- FalseNon hyperfunctioning are hypodense on CT- TruePheochromocytomas are cortical neoplasms- FalseAdrenal carcinomas are medullary- FalseDiscussion-Adenomas are hypodense on noncontrastPheochromocytomas are medullary neoplasmsAdrenal carcinomas are cortical neoplasms35. Usually results in renal failure and need for dialysis?ADPKD- TrueMedullary sponge kidney- FalseMulticystic dysplastic kidney- False36. Medullary cancer of the kidney is associated with?Sickle cell trait- TrueUnder 40 years old- TrueOver 40 years old- FalseSickle cell disease- FalseAfrican american ethnicity- TrueDiscussion-Usually sickle cell trait.Rarely associated with sickle cell disease.37. Wilm’s tumor, which is true?Most common renal mass in children- TrueOften has a pseudocapsule- TrueMostly occurs in 1st year of life- FalsePoor prognosis- FalseMost common abdominal mass in a neonate- FalseDiscussion-Mostly occurs in 6 mo- 4 years with average age of 3 years.Good prognosisMost common abdominal mass in neonates is hydronephrosis38. Acute pyelonephritisKidney enlarged- TrueImmediate persistent dense nephrogram on IVP- TrueCan show dubbing or blunting of minor calyces- TrueTc 99m DMSA shows focal diminished cortical uptake in 90%- TrueDiscussion-Will have a striated nephrogram on CT39. Cause of radiolucent stone?IndinavirDiscussion-Stones caused by anti-retroviral Indinavir are not seen on CT or plain film.Calcium oxalate +/- calcium phosphate- 75%, radio-opaqueStruvite (triple phosphate)- 15%, radio-opaquePure calcium phosphate- 5-7%, radio-opaqueUric acid- 5-8%, radiolucentCystine- 1%, radiolucentIndinavir- <1%, radiolucent40. All causes of bilateral small smooth kidneys except?Acute interstitial nephritis- CorrectNephrosclerosisGeneralized arteriosclerosisChronic glomerulonephritisBilateral renal artery stenosisChronic reflux nephropathy- This could be correct if it didn’t say chronic.Analgesic nephropathy41. SeminomaHypoechoic on US- TrueRadiosensitive- TrueIncreased AFP- FalseRarely metastasizes to retroperitoneal nodes- FalseNot a germ cell tumor- FalseDiscussion-Decreased AFP20% metastasize to retroperitoneal nodesMost common germ cell tumor42. Characteristics of testicular epidermoid?Alternating bands of increased/decreased signal on MRI- TrueIntense enhancement centrally on MRI- FalsePresents as a painful mass- FalseIncreased doppler flow- FalseHyperechoic on ultrasound- FalseDiscussion-PainlessNo enhancementHypoechoic on ultrasound43. Pelvic congestion syndromeInfertility- FalseDyspareunia- TrueVulvar and lower extremity varicosities- TruePelvic heaviness- True44. What causes Meig’s syndrome?Ovarian fibroma/ thecoma- TrueSertoli cellCorpus lutealYolk sacDiscussion-Meig’s syndrome is right pleural effusion, ascites and ovarian fibroma/ thecoma.45. Not a cause of PE?Pelvic congestion syndrome46. TB cystitisUrethra to trigoneTrigone to urethraTrigone to ureteric orifice then moves up superolaterally- CorrectHalf have active TBDiscussion-Most commonly with TB cystitis you will see a shrunken bladder with wall thickening. 47. Ureter injury complication on CT?Penetrating injury is most common- TrueMedial perirenal extravasation with lateral deviation of ureter- TrueRetroperitoneal urine- TrueHydronephrosis of the affected side- FalseOpacification of the proximal portion of the ureter without contrast distally- TrueDilation of ureter distal to site of injury- False48. Adrenal mass with the following Hounsfield units- Unenhanced- 20 HU, Enhanced 80 HU, Delayed- 50 HU. Is it an adenoma?NoDiscussion-Absolute washout= (Enhanced- Delayed) / (Enhanced- Unenhanced)= 0.5Relative washout= (Enhanced- Delayed) / (Enhanced)= -.375 To qualify as an adenoma it must be >0.6 absolute washout or >0.4 relative.49. Not associated with renal cystic disease?Sturge Weber- CorrectZellweger syndromeTuberous SclerosisVHLDiscussion-Sturge Weber- Facial port wine stains and pial angiomas.Zellweger syndrome- Cerebro-heptao-renal syndrome. Death in 1st year.Tuberous scelrosis- Seizures, retardation, adenoma sebaceum, cortical or subependymal tubers, hamartomas, giant cell astrocytomas, renal AML’s, renal cysts, pulmonary LAM, Cardiac rhabdomyomas.Systemic diseases with cysts- tuberous sclerosis, VHL, Meckel-Gruber, Jeune, Zellweger, Conrady, Trisomy 13, Turner50. Most common site of ectopic pregnancy?AmpullaryDiscussion-Ampullary 75-80%Isthmus 10-15%Interstitial 2-4%Ovarian 3%51. What percent of hypertension is renovascular in origin?1-5%52. Papillary necrosis can be seen with?Sickle cellDiabetesAnalgesicsAll of the above- CorrectPapillary necrosis- Pyelonephritis, Obstruction, Sickle cell, TB, Chronic liver disease, Analgesics, Diabetes.53. Most common location of prostate cancer?Peripheral zone- CorrectTransitional zoneCentral zoneDiscussion-Peripheral 80%Transitional 15%Central 5%54. Steinstrasse definition?Discussion-Means “stone street”, it is the appearance of multiple stones lining up in the ureter after lithotripsy.55. Zellweiger syndrome is associated with all of the following except?Seizures- TrueMR- TrueHepatomegaly and jaundice- TrueRenal cysts- TrueBrain dysgenesis- TrueCraniofacial dysmorphisms- TrueDeath in early infancy- TrueMuscular hypertonia- FalseDiscussion-Cerebrohepatorenal syndromeMuscular hypotonia56. Multicystic dysplastic kidney, which is false?Palpable mass- TrueAssociated hypertrophy- FalseDilated tortuous collecting system- TrueNeuro Section1. True of meningiomasWhen extrameningeal they are most common in the paranasal sinuses- FalseWhen in spine, are commonly in the foramen magnum- TrueFrequently hemorrhage- FalseDiscussion-Most common meningeal locations- Falx 25%, Convexity 20%, Sphenoid wing 20%, Olfactory groove 10%, Suprasellar 10%, Posterior fossa 10%.When extrameningeal (less than 1% of all meningiomas),Scalp/skin 40%Ear/temporal bone 26%, Paranasal sinuses 24%.2. True of malignant otitis externaSoft tissue mass at junction of the bony and membranous EAC- TrueTumoral invasion of the temporal bone- TrueMass at the osteochondral junction- TrueIntracranial extent- TrueDiscussion-Is at the junction of bone and cartilage (osteochondral junction)Intracranial extent in 9%3. True of paragangliomasArise from Jacobsen’s nerve- TrueDiscussion- In the jugular foramen, extra-adrenal paragangliomas are called glomus jugulare tumors and arise from either Jacobsen’s or Arnold’s nerve.4. True of glomus jugulare tumorsNodal mets are common- FalseInvolves Jacobsen’s nerve- True15% of people have multiple tumors- FalseHypovascular- FalseBegin in the middle ear and extend into the jugular foramen- FalseDiscussion-Only 2-4 % metastasize20% have multiple paragangliomas.HypervascularThey begin in the jugular foramen and extend into the middle ear.5. True of pituitary lymphocytic hypophysitisSynonymous with pituitary hyperplasia- FalseEnlargement of the neurohypophysis- TrueDiscussion-Rare inflammatory autoimmune disorder with lymphocytic infiltration of pituitary gland.Associated with thyrotoxicosis and hypopituitarism.2 types-Anterior pituitary- lymphocytic adenohypophysitis.Posterior pituitary- Lymphocytic infundibular neurohypophysitis (LINH).Can have enlargement of neurohypophysis if LINH.6. True of traumatic subarachnoid hemorrhageCommon in the interhemispheric fissure- TrueIn acute setting can be diagnosed with FLAIR sequence- TrueCommon in sylvian fissure- FalseCommon in interpeduncular cistern- False7. False of unilateral vocal cord paralysisDilation of ipsilateral vestibule- TrueDilated ipsilateral pyriform sinus- TrueLateral deviation of the ipsilateral arytenoid- False. Flattening of subglottic arch- TrueDiscussion-A dilated laryngeal ventricle is known as the sail sign.The arytenoid is antero-medially rotated.The affected cord will have a paramedian positionThe aryepiglottic fold will be displaced medially8. Normal origin, course and innervation of the 3rd division of the trigeminal nerve.Meckel’s cave, foramen ovale, belly of digastric- TrueCavernous sinus- FalseDiscussion-The mandibular branch of trigeminal nerve is not associated with the?cavernous sinus since it has descended vertically through the foramen ovale underneath the trigeminal ganglion (Gasser ganglion), which is anatomically posteriorly to the cavernous sinus.The 1st and 2nd divisions of the trigeminal nerve do course through the cavernous sinus.9. Arterial supply to posterior thalamusP1- TrueP2- FalseDiscussion-P1- Thalamic and subthalamic arteries supply the posterior medial thalamus.P2- Posterior choroidal supplies the posterior thalamus but not the best answer.10. AV malformationsDevelop over a long period of time- TrueMost present with hemorrhage- TrueUsually supratentorial- TrueDiscussion-85% are supratentorial65% present with hemorrhage which is usually intraparenchymalTrue/ False of NF-1CNS hamartomas- TrueEpendymomas- FalseHamartoma of Iris- TrueCafe au lait spots- TrueDiscussion-NF1- Von Recklinghausen’s NeurofibromasCafe au laitAxillary/ groin frecklingOptic gliomasLisch nodules- pigmented hamartomas of the IrisOther CNS hamartomasSphenoid dysplasia, thinning/dysplasia of long bone cortex.SeizuresNF2- Bilateral acoustic schwannomasMeningiomasEpendymomas12. True of nasopharyngeal carcinomaMost commonly presents as a cervical node13. CADASILInvolves basal ganglia- True (best answer)Involves frontal and parietal lobes and subinsular white matter- TrueDiscussion-Classically involves frontal, temporal and subinsular white matter but can also affect the parietal lobes.14. Canal CN IX passes throughJugular foramen- TrueInternal auditory canal- False15. Not a Posterior DementiaPick’s- CorrectLewy bodyParkinsonsAlzheimersMulti-infarct dementiaDiscussion-Pick’s- Anterior frontal and anterior temporalLewy body and Parkinson’s are in the same spectrumLewy body- Parietal and posterior temporalParkinson’s- Parietal and posterior temporalAlzheimer’s- Parietal and posterior temporal with late frontal involvementMulti-infarct dementia- Not location specificWhich is not a cause of thickened enhancing nerve rootsGuillain barreNF-1Congenital demyelinating diseaseFalse answer not recalledDiscussion-All can cause nerve root thickening and enhancement.Guillain barre- ascending paralysis after infection (campylobacter Jejuni)Spinal Type I dural AVM/ AV fistula Serpiginous enhancing dilated arteries- FalseCord is diffusely expanded and hyperintense on T2- TrueHeterogeneous cord enhancement- TrueFlow voids in the cord- TrueCord atrophy- FalseDiscussion- Type I is most common (80-85%).80-90% in males.Single artery feeder.Enlarged hyperintense distal cord with dilated serpiginous veins (not arteries).Type II is intramedullary AVM60 year old male has a headache and increased intracranial pressure. There is a nonenhancing mass in the frontal lobe that crosses the corpus callosum. Which is the most likely diagnosis?Oligodendroglioma- TrueGBM- FalseLymphoma- FalseGanglioglioma- FalseDiscussion-Crosses midline- GBM, Oligodendroglioma, Lymphoma, radiation necrosis, meningioma, cystic masses, MS.Nonenhancing- low grade astrocytomas, oligodendrogliomas, cystic masses.Oligodendroglioma- 30-50 years old, can cross corpus callosum, subtle ill-defined enhancement in 15-20%. Usually calcified.GBM- by definition is high grade. Can cross midline, usually 65-75 years old, Intense, irregular, heterogenous enhancement of the margins is almost always present.Lymphoma- 30-50 years old, can cross corpus callosum, dense enhancement.Ganglioglioma- Under 30 years old, benign, temporal lobes, seizures, +/- enhancement.Primary CNS lymphomaPeriventricular- True15% of all intracranial neoplasms- FalseDiscussion-Mainly periventricular, may encase ventricles by subependymal spread. Crosses corpus callosum.Stat DX says 6.6% of all intracranial neoplasms.20. Arterial supply to internal capsule, amygdala and hippocampusAnterior choroidal- TruePosterior choroidal- FalseRecurrent artery of Heubner- FalseDiscussion-Anterior choroidal artery Posterior limb of Internal capsule, Amygdala and HippocampusPosterior choroidal arteryHippocampus onlyRecurrent artery of HeubnerBranch of ACA, supplies ant-med caudate, ant-inf internal capsule & putamen.Squamous cell carcinoma of the nasopharynx30% of nasal CA- FalsePresents with cervical LAD- TrueF>M- FalseSmoking association- TrueDiscussion-85% of nasopharyngeal carcinoma.90% initially present because of cervical LADM>F 2.5:1Associated with smokingCongenital cholesteatomaMiddle ear- TrueInternal auditory canal- TrueCochlea- TrueSemicircular canals- ?Osteochondral Junction- False?Diffuse axonal injuryCC involvement denotes a severe initial injury- FalseSevere DAI has a high mortality rate- FalseLower structures/brainstem can be involved- TrueDiscussion-Most common at corticomedullary jx (67%)Adams and Gennarelli stagingStage 1: Frontal and temporal lobe GM/WM interface lesions (mild traumatic brain injury [TBI])Stage 2: Lesions in lobar WM and corpus callosum (moderate TBI)Stage 3: Lesions of dorsolateral midbrain and upper pons (severe TBI)Increasing severity of traumatic force correlates with deeper brain involvementSevere DAI rarely causes death> 90% remain in persistent vegetative state (if brainstem spared)HemangioblastomaPosterior cord with spinal cord cysts and flow voids- TrueDiscussion-Highly vascular tumor of adultsUsually in cerebellum, brainstem and spinal cord.Intra-axial posterior fossa mass.60% cyst with mural nodule. Nodule isointense to brain with flow voids.Associated with VHLWoman with hemorrhage into parasagittal lobes, which is true?Sagittal venous thrombus- TrueTransverse sinus thrombus- FalseCardiac emboli- FalseDiscussion-Parasagittal hemorrhage is highly specific for superior sag sinus thrombosis and is secondary to cortical venous infarction.Occlusion of the recurrent artery of Huebner produces infarct in what structures?Caudate and anterior inferior internal capsuleFalse of Epidural hematomas50% bilateral- False5% mortality- TrueBiconvex- TrueDiscussion- Approximately 15%-20% are bilateral, 5% overall mortality.Which contains the second/maxillary branch of the trigeminal nerve?Superior orbital fissureForamen rotundum- CorrectForamen ovaleForamen spinosumDiscussion-Superior orbital fissure- III, IV, V-1, VIForamen rotundum- V-2Foramen ovale- V-3Foramen spinosum- Middle meningeal arteryOsmotic myelinolysisCan be normonatremic- TrueIn pons <50% of the time- FalseNo enhancement of lesions- TrueDiscussion-In pons a majority of the timeCanavan’s diseaseAffects the subcortical U fibers- TrueMicrocephaly- FalseDiscussion-Spongiform leukodystrophy caused by Aspartoacylase or aminoacylase deficiency.Autosomal recessiveAshkenazi JewsProgressive brain damage from demyelination.Macrocephaly, blindness, paralysis, seizures.Involves subcortical U fibers, Thalami, and globus pallidus.Spares the internal capsule, corpus callosum, caudate and putamen.Which is false about choroid plexus papillomasIn adults it is found in the 3rd ventricle- FalseDiscussion-Adults- 4th ventricle and CPAKids- Trigone of lateral ventricles L>RCan be associated with VHL syndrome.Increased intracranial pressure due to overproduction of CSF.Asymmetric diffuse ventricular dilation= communicating hydrocephalus.Which is due to a neural tube closure defect/ dorsal induction error?Holoprosencephaly- FalseDandy walker- FalseSchizencephaly- FalseCephalocele- TrueDiscussion-Dorsal induction- Cephalocele, Anencephaly, Chiari, DysraphismVentral induction- Holoprosencephaly, Dandy Walker, Callosal agenesisMigration- Schizencephaly, Lissencephaly, PolymicrogyriaTemporal bone fracturesLongitudinal parallels the long axis of the petrous pyramid- TrueDislocation of the ossicles is most common with longitudinal fracture- TrueTransverse is more common than longitudinal- FalseFacial nerve palsy is most common after transverse fracture- TrueDiscussion-Longitudinal- most common, Dislocation of incus or malleus, conductive hearing lossTransverse- CN 8 injury causing vertigo, Facial nerve palsy 50%.Patient presents with purulent otomastoiditis, facial pain, retro-orbital pain and abducens nerve palsy?Gredenigo’s syndromeDiscussion-Petrous apicitis, Most common pseudomonas or enterococcusDeep facial/ trigeminal painMRI shows enhancing mass around petrous apexRamsay HuntDiscussion-Herpes zoster virus. 7th cranial nerve enhancement.Vesicles in external auditory meatus.Similar imaging features to Bell’s palsy.Can also have 8th cranial nerve involvement.Tolosa Hunt SyndromeDiscussion-Intracranial pseudotumor. Granulomatous invasion of cavernous sinus.Unilateral chronic HA. Extra-ocular palsy of 3rd- 6th CN’s.Pain in back of eye.Bilateral facet lock/ dislocation. Which is false?Hyperextension injury- FalseCan have disruption of anterior and posterior longitudinal ligments- TrueCan have disruption of intervertebral disk- TrueCan have disruption of Sharpy’s fibers- TrueDiscussion-Hyperflexion injuryOrbital pseudotumorHypertrophy of the extraocular muscles only- FalseUnilateral is most common- TrueEnlarged lacrimal glands- TrueProptosis- TrueUvoscleral thickening and enhancement- TrueEnhancement of optic nerve sheath- TrueDiscussion-Lymphocytic infiltration and hypertrophy of all intraorbital soft tissues.Graves only affects the muscles and fat.Myxopapillary ependymomaConus medullaris- Correct4th ventricleLateral ventricleCervical cordDiscussion-Most common conus tumor (83%). Ependymomas are most common intramedullary neoplasm in adults.Cervical > Thoracic > Conus.Intense enhancementEmbryologic origin of cerebral hemispheres?TelencephalonDiscussion-Telencephalon- Cerebral hemispheres, putamen, caudateDiencephalon- Thalamus, hypothalamus, globus pallidusMesencephalon- Brainstem above pons, cerebral peduncles, colliculiMetencephalon- CerebellumMyelencephalon- Medulla, ponsCause of atrophy of the cervical cord and cerebellum?Freidrich’s ataxia- CorrectEtOH toxicityOlivopontine degenerationDiscussion-Freidrich’s ataxia- cerebellar and c spine atrophyEtOH causes cerebellar atrophy/ vermal atrophyOlivopontine is cerebellum without c spine involvementNot associated with septo-optic dysplasia (DeMorsier syndrome)?Retinal aplasia- CorrectCallosal agenesisHypothalamic hypopituitarismSchizencephalyAbsent septum pellucidumBubbly appearing mass in lateral ventricle with signs of increased ICP. What is the most likely diagnosis?Central neurocytoma- CorrectGBMEpendymomaMeningiomaL4-5 disk herniation is most likely to affect which exiting nerve?L5False of enthesioneuroblastoma?Rarely intracranial- FalseArises from olfactory villi- TrueBimodal age distribution- TruePresents with epistaxis and nasal obstruction- TrueDiscussion-Arises from olfactory epitheliumYoung men and 50-60 years old- bimodalOther symptoms include headache, visual changes.Cysts along superior tumor margin in anterior cranial fossa (intracranial).Large frontal lobe mass with calcifications and edema?Oligodendroglioma- CorrectGBMDiscussion-Slow growing tumor is usually large at presentation.90% have microcalcifications.Also have large nodular calcifications.50-65% in frontal lobes.Minimal edema.Post viral syndrome, abnormalities in brain and cervical cord?ADEM (acute disseminated encephalomyelitis)Discussion-Post-infection encephalitis 7-14 days post viral/vaccination. Multifocal subcortical, asymmetric WM abnormalities with sparing of cortical gray matter. Can involve brainstem/posterior fossa. Can enhance, no progression of lesions on follow-up. Septic patient with atrial fibrillation and peripheral brain aneurysms?mycotic aneurysmsDiscussion-3% of all aneurysms65% due to subacute bacterial endocarditisPeripheral location49. Leigh’s disease?Commonly seen in adults- FalseIncreased T2 signal in basal ganglia- TruePoor suction hypertonia- TrueDiscussion-Chronic basal ganglia lesion in childhood, due to inborn error of metabolism. Subacute necrotizing encephalomyelopathy. Autosomal recessive. Propensity for putamen.Regarding capillary telangiectasiasSeen on angiography- FalseEnhance on MRI- TrueMost common in pons- TrueSeparated by normal brain parenchyma- TrueRegarding hyperparathyroidismSecondary is associated with MEN syndromes- FalseBrown tumor is more common in primary but is seen more frequently in secondary- TrueSoft tissue calcification is seen more commonly in secondary- TrueDiscussion-Primary is associated with MEN 1 and MEN 2MEN 1- hyperparathyroid, pituitary tumors, Islet cell tumors (gastrinoma, glucagonoma).MEN 2- hyperparathyroid, Pheochromocytoma, Medullary thyroid CA.Primary is associated with brown tumorsSecondary is associated with soft tissue calcs52. Which of the following is false regarding vasogenic vs. cytotoxic edema?Infarct will ultimately have both types- TrueTumors are vasogenic- TrueVasogenic will “frond-like projections” into the white matter- TrueTransependymal interstitial edema is CSF- TrueThe edema causes shortening of the relaxation times of T1 and T2- FalseDiscussion-Vasogenic- breakdown of BBB with sparing of gray matter.Cytotoxic- Nonfunctioning Na and Ca pumps, affects white and gray matter.Infarcts start with cytotoxic edema but eventually have vasogenic edema as well.Edema causes lengthening of T1 and T2 relaxation times.Clival tumor 40 year old male with slow developing opthalmoplegia?ChordomaLateral medullary syndrome (Wallenberg syndrome) results from infarct of?PICADiscussion-Lateral medullary syndrome (Wallenberg syndrome) classic crossed face and body symptomsIpsilateral Horner (ptosis, anhydrosis, miosis), ipsilateral face loss of pain and temperature, contralateral body loss of pain and temperatureContralateral extremity ataxiaMay have dysphagia, dysarthria, vocal cord paralysis from CN10, nausea, vertigo, hiccupsTreatment often supportive, IA or IV thrombolysis if associated with additional vertebrobasilar occlusionCarbon monoxide poisoning?Lentiform nucleus and globus pallidusWhich brain met is not hemorrhagic?Esophageal- CorrectMelanomaChoriocarcinomaRenal cellThyroidPosterior effacement of third ventricle?Germinoma- CorrectColloid cystDiscussion-Germinoma (pineal) posterior effacementColloid cyst anterior effacementEpidural hematomas5% bilateral- TrueBiconvex- TrueCross sutures- False95% associated with fracture- TrueCreutzfeldt Jakob disease on MRIT2 hyperintense in basal ganglia, thalamus and cerebral cortex.EpendymomasThird most common posterior fossa tumor in peds- True50% calcify- TrueCan have dropped mets- TrueHypodense on CT- FalseDiscussion-Hyperdense or heterogeneous on CTLocation of ectopic thyroidNeckTongue and mediastinumPelvisAll of the above- CorrectEarly subacute hemorrhage on MRIBright on T1, Dark on T2Discussion-Hyperacute- T1 iso, T2 brightAcute- T1 iso, T2 darkEarly subacute- T1 bright, T2 darkLate subacute- T1 bright, T2 brightChronic- T1 dark, T2 darkNuclear Medicine Section1. Nuclear medicine cystogramCan be used to quantify residual- TrueCan detect as little a 5 ml reflux- False50-200 times less radiation than contrast cystography- TrueMore sensitive for reflux evaluation- TrueDiscussion-Can detect as little as 1 ml reflux.NM cystogram sensitivity 91%, VCUG sensitivity 45%.2. Not a correct nuc med relationshipDTPA renal cortical abnormalityDiscussion-DMSA is for evaluating morphology/ renal cortical uptakeDTPA for glomerular filtration and used to calc GFR.Mag 3 is for tubular secretion and estimates ERPF effective renal plasma flow and gives better images.3. Renal artery stenosis with captopril study?Poor renal function curve after captopril given- TrueCortical retention- TrueCaptopril increases GFR in affected kidney- FalseDiscussion-Ace inhibitor reduces GFR therefore have cortical retention.4. Not a cause of superscanATN- CorrectRenal failureMetabolic bone diseaseMetsDiscussion-ATN has increased renal uptake so would not look like a superscan.In a superscan you should not see much renal uptake because other regions are using up all of the radiopharmaceutical.In renal failure it would look the exact same as a super scan except you will not have any uptake in the bladder as no urine was produced.Other causes of superscan-diffuse mets, renal osteodystrophy, osteomalacia, hyperparathyroidism (usually secondary), hyperthyroidism, Myelofibrosis, mastocytosis, Paget’s.5. Nuclear medicine relationships- correct or incorrectI-125: Thyroid cancer- FalseSamarium 153: refractory lymphoma- True32-P sodium phosphate IV: polycythemia vera- TrueOctreotide: somatostatin receptor- True32-P colloid and mesenteric intracavitary lesion- TrueDiscussion- Iodine 131- Thyroid ablationIodine 125- Usually for prostate and brain brachytherapy but can also image the thyroid.Iodine 123- Thyroid imagingFalse regarding FNH enhancement patternNormal to slightly increased uptake with persistent enhancement on HIDA- TrueCan be photopenic to hot on sulfer colloid- TrueIs hot in the quadrate lobe- FalseDiscussion-First answer choice may have been worded late filling?7. All are uses for radionuclide cisternography except?Ventricular shunt patency- TrueCSF leak (CSF rhinorrhea or otorrhea)- TrueSuspected NPH- TruePorencephalic cyst, leptomeningeal cyst or posterior fossa cyst- TrueEvaluate for parasaggital ascent of radiotracer in 24 hours- FalseDiscussion-Activity in basilar cistern should be seen at 2-4 hours.Activity in vertex at 24- 48 hours.Ventricles should not normally be seen at any time.Basilar cisterns should be clear by 24 hours.Minimal activity in lateral ventricles may be normal in older patients.NPH shows early ventricular visualization which persists for 24-48 hours.8. Causes of a false positive HIDA (Alternate recall asked which is most likely)Acalculus cholecystitis- FalsePancreatitis- TrueAscending cholangitis- TrueProlonged fasting- True (most likely cause)Discussion-Acalculous cholecystitis causes a false negative.Cause a false positive on HIDA- Pancreatitis, Ascending cholangitis, Fasting, TPN, Alcoholism, Narcotics, cystic duct obstruction, liver dz, severe chronic cholecystitis.9. Not a cause of false negative HIDAAcalculous cholycystitisDilated cystic duct- CorrectDuplicated common cystic ductDiscussion-Causes of false negative HIDA-Acalculous cholecystitis, duplicated cystic duct, duodenal divertic.Ga-67 lung scan does not show increased uptake in?Sarcoid- FalseBleomycin toxicity- FalsePCP- FalseIPF- FalseDiscussion-Positive Gallium scans with all of them.Gallium scans look for pulmonary inflammation.Infection- TB, PCP, CMVInflammation- Sarcoid, Interstitial lung dz (Pneumoconiosis, IPF, lymphangitic carcinomatosis), Radiation phaseDrugs- Bleomycin, AmiodaroneBest to identify splenic tissue/ splenuleTagged RBC- FalseSulfur colloid- TrueTc99 Pertechnetate- FalseHIDA- FalseDiscussion-Best modality would be heat damaged tagged RBC scan.Sulfur colloid works but not nearly as well as heat damaged tagged RBC scan.I-131 treatment given to wrong person, which is incorrect.Misadministration- TrueReportable- TrueMust call NRC within 24 hours- False?Discussion-Possibly 48 hoursNot PET avidBAC- TrueSquamous cell lung CAAdenocarcinomaSmall cell carcinomaDiscussion-BAC PET negative in 55%False negatives in PET- BAC, carcinoid, mucinous adeno, lobular breast, RCCStripe sign on VQ scanVery low probability- TrueIndicates COPD- TrueHigh probabilityIndeterminateDiscussion-Stripe sign- Band of normally perfused lung interposed between a perfusion defect and adjacent pleural surface. Indicates COPDTwo or more moderate/large segmental mismatch defects on VQ scan has high probability per PIOPED criteria.Fissure Sign - decreased line of perfusion due to fluid in fissureTriple Match - V, Q and CXR, often intermediate probabilityWhole lung perfusion defect - pulmonary agenesis, hilar mass, medistinal fibrosisWhich is more sensitive for GI bleed?Tagged RBC scan- TrueCT angio- FalseDiscussion-Can detect GI bleed as little as 0.1 mL/minWith sestamibi parathyroid scan, do a delayed image 2 hours later to differentiate?Thyroid adenoma from thyroid cancer- FalseParathyroid adenoma from parathyroid cancer- FalseParathyroid nodule from thyroid tissue- TruePhotogenic area in liver on HIDA scan. On 2 hour delayed images, the region fills in. This is most consistent with?Atypical hemangiomaHepatoma/ HCCFNH- TrueDiscussion-Hepatocytes take up HIDA.Kupfer cells take up Sulfer Colloid.Atypical hemangioma- Neither hepatocytes or Kupfer cells, just tangled vessels. Cold on HIDA and SCHepatoma/ HCC- Dysfunctional hepatocytes only. Usually no uptake/cold spot on both HIDA and Sulfer colloid.FNH- Hepatocytes and Kupfer cells. Normal to increased uptake with delayed persistant uptake due to dysfunctional biliary canaliculi on HIDA and on SC.False regarding Meckel’s diverticulum scan50% contain ectopic gastric tissue- TrueCan premedicate with pentagastrin for better visualization- TrueCan premedicate with H2 blocker for better visualization- TrueCan premedicate with glucagon for better visualization- TrueCan premedicate with perchlorate for better visualization- FalseStomach takes up 25% of administered dose- TrueDiscussion-50% of symptomatic patients have ectopic gastric tissue within Meckel’s.Rule of 2’s- 2 ft from ileocecal valve, under 2 years old, 2% of population, length of 2 inches.Pentagastrin 20 min prior to Pertechnetate increases uptakeCimetidine 1 hour prior to Pertechnetate decreases secretionGlucagon 10 min prior to Pertechnetate decreases peristalsisPerchlorate suppresses localization and should be avoided.Two large segmental defects without match on ventilation images on a VQ scan. What is the probability of PE?LowIntermediateHigh- TrueThyrogenDecrease symptoms of hypothyroidism- TrueIncrease uptake for ablation therapy- TrueDiscussion-Normally patients have to stop taking T3/4 prior to ablation and will have symptoms of hypothyroidism. Thyrogen is a TSH analog which is used to make the thyroid hungry for iodine prior to ablation. It also allows them to continue taking synthetic T3/4 and not have the symptoms of hypothyroidism.21. Incorrect regarding sestamibi breast scan2:1 counts in malignancy vs. normal parenchyma- FalseInject feet if suspect bilateral malignancy- TrueSupplanted by biopsy- TrueMisses small cancer- TrueDiscussion-6:1 counts in malignancy vs normal parenchymaInject contralateral arm if unilateralMisses small cancer < 1cm and misses deep cancer.Which has not demonstrated increased activity on MDP bone scan?Multifocal pneumonia- Correct MITissue infarctMalignant effusion/ ascitesSplenic infarctCerebral infarctDiscussion-Any tissue infarct, malignant ascites, dystrophic calcs, dermatomyositis, and amyloidosis will show increased uptake.23. False regarding HIDA scan in a newborn with jaundice? Activity in the bowel exclude hepatitis- False? Life of Tc99 allows for 24hr delayed imaging- FalseActivity in bowel excludes biliary atresia- TrueActivity in GB excludes cholecystitis- TruePretreatment with Phenobarb can help with neonatal hepatitis and hepatic disorders- TrueCannot differentiate between severe neonatal hepatitis and biliary atresia- TrueDiscussion-Differentiating between biliary atresia and neonatal hepatitis: if there is activity in the extrahepatic biliary system, there is no biliary atresia.In severe liver disease there may not be activity at 24 hours so you need to do 24hr delayed images, but can’t do with Tc99m.The phenobarbital will minimize false positives in those with patent biliary systems but poor secretion by increasing secretion from the liver into the biliary system.PET/CT not good for detection of?Mucinous adenocarcinoma of colon- TrueNSCLC (BAC shows low uptake)Melanoma LymphomaDiscussion-Mucinous Adenocarcinoma has low/variable uptake.NSCLC, BAC has low uptake but this is not specific for BAC.Melanoma has avid uptake if lesion is big enough.False negatives in PET- BAC, carcinoid, mucinous adeno, lobular breast, RCCFDG-PET is least sensitive for which of the following?Colon CALymphomaProstate CA- TrueEsophageal CA26. Hibernating myocardiumFixed perfusion defect- TrueDiscussion-Myocardial cells can hibernate when exposed to ischemia without infarct. If reperfused by stenting or ionotropic agents, they can regain normal function over time.27. Which is not true concerning HDP bone scan?HDP is a calcium analogue- TrueTechnetium dose is 3-5 mci- FalseDiscussion-HDP- hydroxymethylene diphosphonate, calcium salt analogueDose is approximately 20 mci28. Which radiotracer does not normally demonstrate bowel activity (D: 1101)Indium- CorrectGalliumDiscussion-Indium has no intestinal/urine activity.Gallium imaging is done at 48-72 hours after background bowel activity has decreased. In abdominal imaging of acute infection/ abscess, which agent is preferred?Indium 111- CorrectGallium 67Tc 99m PertechnetateTc 99m Sulfur colloidDiscussion-Indium > Gallium > Tc 99m for abscess/infection.Benefits of Indium over Gallium include all of the following except?Later results- FalseBetter distribution over tissue- TrueStable distribution- TrueLower dose in non-infection tissue- TrueDiscussion-Earlier results with Indium- Image at 18-24 h with Indium, 48-72 h with GalliumWhich agent is used to study brain death?DTPACeretec- TrueDiscussion-Ceretec is Tc 99m labeled HMPAO (hexamethylpropylene amine oxime)DTPA can also be used but is not as specific.32. Regarding SPECT brain imagingCeretec distribution is proportional to blood flow and crosses the blood brain barrier with predilection for cortical gray matter- TrueCan be utilized to determine brain death- TrueAbnormal temporal and parietal lobe activity in Alzheimer patients- True Used in determination of acute stroke, TIA, seizures and dementia- TrueCan’t differentiate radiation necrosis vs recurrence- FalseDiscussion-If Thallium is used, it is useful in differentiating necrosis vs recurrence.33. Regarding FDG PET in oncologic imaging, which malignancy is imaged prior to, and immediately following treatment to direct further management?Breast CA- CorrectColon CA Thyroid CAThallium uptake can be seen in which of the following?Breast CAThyroiditisExercise induced skeletal muscleAll of the above- CorrectDiscussion-Thallium is taken up by cells with active NA/K ATPase and is proportional to regional blood flow.Gallium 67 is not normally taken up in?LiverHeart- CorrectBreastColonLacrimal glandDiscussion-Uptake in RES, liver, spleen, bone marrow, bowel/ colon, renal cortex, nasal mucosa, lacrimal/salivary glands, blood pool, breasts. NOT taken up by hemangioma, benign neoplasms, cirrhosis, thyroid, cystic disease of breast, reactive LAD.Which is incorrect regarding Gallium scintigraphy?PCP increases gallium uptake- TrueGallium is not taken up in Kaposi sarcoma- TrueIn PCP, the lungs demonstrate more uptake than the liver- TrueIn patients on pentamidine prophylaxis, gallium is redistributed to upper lobes- TrueDiscussion-Gallium imaging in PCP: Increased bilateral uptake, generally decreasing with treatment. There is redistribution in the lung apices as treatment progresses due to accumulation of abx in the bases. It does not show uptake in Kaposi’s, and does not depend on pyogenic response (good for immunocomp). Uptake is greater than or equal to liver.Do kidneys have increased uptake of Tc 99m MDP after chemotherapy?YesWhich is false regarding V/Q scan?Reduce MAA particles in pulmonary arterial hypertension- TrueBiologic half life of Tc 99m MAA is 12-15 hours- FalseRetention of radiotracer activity in ventilation phase following administration of pentamidine- TrueUnderlying parenchymal disease can result in matched defect- TrueDiscussion-Half life of Tc 99m MAA is 6-8 hours.In hibernating myocardium, FDG-PET is usefulIn patients with normal Thallium 201 studyWhen perfusion is normalIn evaluation of a patient 4-6 months following MI- TrueDiscussion-Useful in patients with abnormal Thallium and perfusion studies to determine viability of myocardium.FDG used by ischemic but viable myocardiumH or Honda signSacral insufficiency fracture1 cm solitary pulmonary noduleUse FDG- PET to determine benign or malignant.Best way to diagnose chronic acalculous cholecystitis?HIDA with CCKCause of stripe sign on V/Q scan?Non-embolic cause or old/ resolving PETc 99m Mag-3 is better than DTPA for which of the following?Renal hypertensionPolycystic kidney diseaseAll of the above- TrueDiscussion-DMSA is for evaluating renal cortical uptakeDTPA for glomerular filtration and used to calc GFR.Mag 3 is for tubular secretion and estimates ERPF effective renal plasma flow and gives better images.45. Solitary liver lesion that is hot on liver scan?FNH- TrueHCCHepatic adenomaHemangiomaDiscussion-Hot spot: FNH, regenerating nodule in cirrhosis. SVC obstruction hot spot is in left hepatic lobe (umbilical vein). Budd-Chiari hot spot is in caudate lobe with decreased activity in the rest of the liver.Gallbladder EF in HIDA scan<35% is abnormal- TrueDiscussion-Must be related to clinical findings<30% is indicative of chronic cholecystitis35%-50% is borderline.Patient with bone mets shows increasing activity on bone scan 3-6 months after treatment. Flair phenomenon- good prognosis/ response.Not a cause of superscan?Prostate metsBreast metsMultiple myelomaPrimary hyperparathyroidism- CorrectDiscussion-Both primary and secondary can cause superscan but typically secondary.Indium 111 can be used for?Diverticulitis- FalseBowel inflammation- FalsePhlegmon- TrueDiscussion-Indium for infection/abscess/phlegmonInterventional / Vascular Section1. Fibromuscular dysplasia: % that recover/improve/resolve after angioplasty90%- True70%Discussion-Stat Dx says >95% improve after angioplasty.2. Brodel’s lineAvascular plane in the kidneyDiscussion-Brodel’s line is posterior lateral convex border of kidney.Good for biopsies.3. Signs of aortic dissection with arch angiogramCentral deviation of the catheter- FalseAortic regurgitation- TrueEarly dense filling of the false lumen- FalseFalse lumen larger than true lumen- TrueDiscussion-Dahnert 619. Lateral deviation of catheter.Aortic regurgitation in 30%.Dense filling of true lumen.4. Where does a TIPS clot offThe hepatic veinsIn the graft- TruePortal veinDiscussion-Most common place for stenosis is at hepatic vein end of stent- Stat DX.It seems like the clot would then be within the stent.5. Absolute contraindication to TIPSSevere systemic infection- FalseSevere cirrhosis- TrueSevere left heart failure- FalseBudd Chiari- FalsePortal vein thrombosis- FalseDiscussion-Absolute contraindications are-Severe R heart failure, Polycystic liver, Severe encephalopathy, severe cirrhosis.Relative-Portal vein thrombosis, severe systemic or intraabdominal infection.6. Least likely place for clot to form in dialysis graftCentral veins- TrueWithin the graft- FalseVenous outflow/ anastamosis- FalseArterial inflow/ anastamosis- FalseDiscussion-The most common site in prosthetic arteriovenous grafts for clot or stenosis to occur is at the anastomosis between the graft and outflow vein, as identified in 47% to 60% of cases, or in the outflow vein itself in autogenous arteriovenous fistulas.The least common place for clot or stenosis would be in the central veins.7. True of RCC biopsyRisk of seeding the tract no longer an issue with current biopsy tools- TrueUsed in people with comorbid conditions- True8. Normal ABI values0.9 – 1- True0.8 – 0.90.7 -0.80.6 – 0.79. Regarding a duplicated IVC usually drains (Alternate recall- Regarding left sided IVC)Usually drains to left renal vein- TrueDrains into hemiazygous- False10. False lumen in abdominal aortic dissectionAnterior left- Posterior left- TrueAnterior right- Posterior right-Discussion-Ascending aorta dissection is anterior right and it twists to be posterior left in descending aorta.Follows the course of the aorta (it ascends anteriorly on the right and descends posteriorly on the left.11. True of aortic intramural hematoma on contrast enhanced CT Hyperdense, non-enhancing, flapHyperdense, non-enhancing, no flap- TrueDiscussion-Typical absence of intimal tear/flap (Stat Dx).12. Which arises from the posterior branch of the internal iliac arteryPudendal- FalseInferior Gluteal- FalseIliolumbar- TrueObturator- FalseDiscussion-Anterior branches of internal iliac Obturator, inf gluteal, umbilical, uterine, inf vesicle, mid rectal, int pudendalPost branches of internal iliacIliolumbar, lateral sacral and superior gluteal.13. False of anomalous coronary arteriesThe interarterial is worrisome and requires surgical correction- TrueThe retroaortic type is ominous- FalseAround 1% of the population- TrueMost are benign- TrueLCA can originate from RCA- TrueDiscussion-Interarterial is malignant (needs surgery). Goes between aorta and pulm artery.Occurs in 0.3- 1.3% of population.Regarding uterine artery embolizationCan reattempt if not much result first time- TrueBetter for large lesions- False Not painful- FalseSuccess rate 80-85%- Probably trueThe uterine artery typically arises from the posterior division of the internal iliac artery- FalseDiscussion- Often reattempted if not adequate results first time.Larger lesions often have multiple arterial supplies and sometimes have central necrosis, both make embolization less likely to work.Pain is most common side effect of embolization and is often severe.Success rate is around 94%.Uterine artery arises from the anterior division of the internal iliac artery.Long segment narrowing of the SMA in a 20 year old female.TakayasuDiscussion-Takayasu’s is a medium and large vessel vasculitis.Especially aorta and it’s branches.Can get aneurysms and stenosis.Which should not be stented?Fibromuscular dysplasiaAortic dissection with decreased flowOcclusion of the artery proximal with distal thrombosis- MaybeDiscussion-You can stent fibromuscular dysplasia.You can stent aortic dissection with decreased flow.IVC filter placement indicationsFree floating DVT in femoral or iliac veins- TrueIntracranial hemorrhage on anticoagulation- ?Gastric bypass- TrueProximal free floating DVT is relative indicationSecond choice was probably incorrectly recalled?Bariatric patients undergoing surgery for weight control BMI >55 is relative indication and is controversial.AV malformations20% bleed per year- False10-15% are associated with Osler Weber Rendu syndrome- ?Spontaneously regress- MaybeDevelop over time as opposed to being congenital- MaybeAre more common in posterior fossa- FalseDiscussion-2-4% risk of rupture per year. (statdx)Can spontaneously regress after rupture in very rare cases.AVM’s are congenital but they also develop over time.85% supratentorial, 15% posterior fossa. Most common cause of AAAASVDVasculitisTraumaHypertensionDiscussion-ASVD causes 95% of AAA’sBuerger’s disease90% involve upper extremity- FalseAssociated with heavy smoking- TrueMost commonly occurs in young males- TrueLower extremities are most common- TrueDiscussion-Thromboangiitis obliterans, affects arteries and veins.Legs 80%Arms 20%Most common in males between 20 and 40 y.o.Sturge WeberChoroid plexus angioma- TrueHemangioblastoma- FalsePort wine stain- TrueHemiatrophy of cortex with calcs- TrueDiscussion-Hemangioblastoma is with VHLSturge Weber- Port wine stain, pial angiomas, seizures, hemiparesis.Takayasu arteritisPts are >50 years old usually- FalseInvolves the SMA and pulmonary arteries- TrueMost common in males- FalseDiscussion-Patients are typically <50Most common in females M:F 1:8All are indications for TIPS except?Severe hepatic failure- FalseRefractory ascites- TrueHepatic hydrothorax- TrueBudd Chiari- TrueDiscussion-Indications for TIPS-Intractable ascitesVariceal bleedingBudd ChiariWhich is a branch of the superior mesenteric artery?R gastric- FalseMiddle colic- TrueProper hepatic- FalseGastroduodenal- FalseInferior pancreaticoduodenal- TrueInferior rectal- FalseDiscussion-Branches of Celiac- Left gastric, common hepatic (proper hepatic, right gastric and gastroduodenal are branches of this), splenicBranches of SMA- Inferior pancreaticoduodenal, intestinal, ileocolic, R colic, middle colicBranches of IMA- L colic, sigmoid, superior rectalInferior rectal artery arises from the internal pudendal artery which is a branch of the anterior division of internal iliac artery.Most common primary vascular tumorAortic sarcomaPulmonary sarcomaLipomyosarcomaLeiomyosarcoma- CorrectDiscussion-When considering primary vascular tumors, most are malignant and are much more common in veins.Leiomyosarcoma is the most common intravascular venous tumor.Embryonic remnant between Celiac and SMA-Arc of Buehler- CorrectArc of BarkowArc of DrummondArc of RiolanDiscussion-Barkow- R and L gastroepiploicArc of Drummond (Marginal artery of Drummond)- SMA (ileocolic, R colic, mid colic) to IMA (L colic and sigmoid branches)Arc of Riolan- SMA (mid colic) to IMA (L colic)Vessel involved in median arcuate ligament syndrome?Celiac arteryDuring angiographic procedure the patient’s blood pressure drops and heart rate goes down. The patient is diaphoretic and confused. What happened?Vasovagal reaction- CorrectInternal hemorrhageMIContrast reactionRegarding radiation safety, which is False?Causes cataract and thyroid cancer in the operator- TrueLead shielding decreases exposure- TrueTIPS and UAE have the highest exposure- TrueFluoro time is accurate in determining skin exposure- FalseWhich is not a cause of central venous occlusion?Subclavian steal- CorrectBronchogenic cancerRadiation fibrosisDiscussion-Subclavian steal is arterial occlusion in the subclavian artery just proximal to the vertebral artery origin. This causes reversal of flow in the vertebral artery in order to supply blood to the arm and at the expense of the vertebrobasilar circulation.Indications for draining a pancreatic pseudocyst?Suspected infection- TrueCausing GI or biliary obstruction- TrueIncreasing size or pain- TruePersistence of pseudocyst >5 cm- TrueCan traverse colon if lesion is cystic- FalseDiscussion-Can traverse stomach but not colonKnow types of endoleaksType 1 = incomplete fixation of stent-graft to aortic wall at the proximal/distal attachment siteType 2 = retrograde flow via parent artery MC (lumbar / inferior mesenteric artery)Type 3 = endograft defect with disruption of either metallic support or fabricType 4 = design failure, porous graft- bleeding thru graftType 5 = endotension, no visible contrast or flow within aneurysm sac but continued expansionWhich is not a compression syndrome?Leriche- CorrectThoracic outletMay ThurnerHypothenar hammer syndrome?Ulnar artery is affected- TrueIs related to Guyon’s canal- TrueAffects the median nerve- FalseDiscussion-Repetitive trauma to hypothenar eminance causes stenosis or occlusion of ulnar artery and/or proximal superficial palmar arch.Ulnar nerve and artery go through Guyon’s canalTrue of aortic aneurysmsIncreased risk if first degree relative with AAA- True90-95% are infrarenal- TrueCrescent sign indicates impending rupture- TrueMost commonly under 65 years old- FalseDiscussion-10 x increased risk if 1st degree relative with AAACrescent sign- acute intramural hematoma, impending ruptureMost common >60 years old.Bland-White Garland syndromeLeft coronary artery from pulmonary artery.Most common symptomatic Aortic arch anomaly?Double arch- TrueRight arch with aberrant L subclavLeft arch with aberrant R subclavMirror R archDiscussion-Most common symptomatic type is Double archDouble arch- anterior tracheal and posterior esophageal impressions.Also on AP will have high R and lower L esophageal impressions.Right arch with aberrant L subclavian- second most commonR sided and posterior esophageal impressions.Left arch with aberrant R subclavian-L sided and posterior esophageal impressions.Most appropriate location for the femoral stick on angiogram?Above inguinal ligament- FalseBelow inguinal ligament in SFA- FalseBelow inguinal ligament at level of femoral head- TrueDiscussion-The common femoral artery is accessed at the level of the femoral head.Radiofrequency ablation, which is false?Insulated probe with active tipAblating next to a vessel is good because of the ? effect- FalseDiscussion-Don’t ablate next to vessel as it carries away heatChest biopsy, which is false?Can use ultrasound- TrueFluoroscopy is adequate in most cases- FalseFNA and biopsy have 70-79% sensitivity for cancer- TruePlace biopsy site up to prevent complications- TrueBreast Section1. True/False of PapillomaIntraductal subareolar mass most common region- TrueDilated ducts- TrueSolid intraductal mass- TruePresents as bloody nipple discharge- TrueMultiple filling defects are called papillomatosis- TrueIncreased risk of invasive spread if multiple and peripheral- TrueMRI can make the diagnosis- FalsePresents similar to Paget’s- FalseAppears as an irregular nodule on mammogram- FalseDiscussion-Usually papillomas present with bloody nipple discharge.Paget’s presents with skin changes, 50% palp mass, bloody discharge.Usually not seen on mammogram. If seen, round or oval nodule, not irregular.MRI not specific, need galactography.Breast lesion which is hypoechoic on ultrasound and MRI shows bright T2 with peripheral enhancement.DCIS- FalseInfected cyst- True3. ACR indications for breast MR screeningGreater than 30 percent lifetime risk breast cancer- FalseDense breasts- FalseStrong family history- TrueDiscussion-Greater than 20% lifetime risk is an indicationYoung patient with +BRCA mutationPalpable mass with negative mammo and ultrasoundDense breasts are not an indication alone but they are if in conjunction with high risk lesion of LCISStrong family history is an indicationAxillary node malignancy with negative mammoPlanning for biopsy to determine scar vs malignancyStaging of tumor4. Multiple filling defects on ductogramDuctal ectasia- FalsePapillomatosis- True5. Inflammatory breast cancerCan present as thick edematous breast- TrueInitial treatment involves antibiotics- FalseDiscussion-Although initial treatment of breast erythema would involve antibiotics, initial treatment of inflammatory breast cancer is radiation therapy.Need core biopsy. Often mammographically occult.6. True/false sebaceous cystNot palpable- FalseThick draining material- FalseLocalizable to the skin- TrueUltrasound if subcutaneous- TrueDiscussion-Epidermal inclusion cysts have thick draining material but not sebaceous cysts.7. Best to evaluate skin calcsTangential view- True8. Well circumscribed soft tissue density that turns out to be breast cancer. Age 50Medullary breast cancer- True Invasive ductal NOS- FalseLobular- FalseDiscussion-Medullary is a type of IDC (invasive ductal cancer) and is a well circumscribed noduleSee question 14.9. True of Paget’s disease of breastTreat with local radiation- FalsePresents with skin/areolar ulceration and bleeding- TrueNeed core biopsy, punch biopsy not good enough- FalseAssociated with underlying ductal CA- TrueDiscussion-Few select cases can receive local radiation therapy or post breast conserving surgery radiation, but standard of care is mastectomy.Skin ulceration and bleeding only in advanced disease.Typically present with pruritis and scaling and erythema.10. True of Hormone replacement therapySignificantly lowers sensitivity of mammo- TrueCauses diffuse enhancement on MR- TrueTakes years to develop denser breast tissue- FalseIncreases the density of breasts- TrueSkin thickening and architectural distortion is most common complication- FalseDiscussion- The dense breast tissue can develop within weeks.11. Essential features of mammo submitted to ACR for evaluation/ DMQSA mandates.Nipples in profile with nipple markers- TrueNo artifacts- FalsePectoralis major muscle visible at the level of the nipple on MLO view- TrueDiscussion-Essential- nipple in profile/ positioning, compression.No artifacts is technically not essential according to ACR but recommended.12. True regarding breast mammo mass and densityA mass is space occupying seen in at least 2 views- TrueDensity is seen in at least one view- FalseBoth seen on two views- FalseDiscussion-Lexicon doesn’t use “density”, they say asymmetry.Mass- 2 views, Asymmetry- 1 view.True/False of male breast cancerAssociated with Klinefelter’s syndrome- TrueAssociated with gynecomastia- FalseUsually in upper outer quadrant- FalseUsually presents as painless subareolar mass- TrueInvasive lobular carcinoma is rare- TrueAssociated with BRCA-1 mutation- FalseNot treated the same as female breast CA- FalseDiscussion-Klinefelter’s 47 XXY, 6% of male breast cancer, 3% lifetime risk.Not associated with gynecomastia.Most common location is subareolar.Invasive lobular carcinoma is extremely rare- 1-2%.Most common type is infiltrating ductal.Associated with BRCA-2Which lesion is associated with microcalcifications 70% of the time?LCIS- FalseDCIS- TrueInvasive ductal- FalseInvasive lobular- FalseDiscussion-LCIS 10% have microcalcs. Arises in epithelium of mammary lobules. Not precancerous but marker of 10 fold increased risk of subsequent invasive cancer in either breast. Most common bilateral tumor 30-67% (statdx).Non-palpable and radiographically occult.DCIS 10% of all breast cancers. 72% have microcalcs. High nuclear grade (comedo type), necrotic, 90% calcified. Low nuclear grade (noncomedo type), non-necrotic, 50% calcified. Psammoma like bodies. Fine or coarse granular calcs. “arrowhead calcs” (Nick likes this one). 30% become invasive after 10 years. 10% are palpable.IDC Overall 65% of breast cancers. Invasive ductal carcinoma 45-60% have microcalcs? How can invasive have fewer calcifications than DCIS when it started as DCIS? Medullary is a special type of IDC and is well circumscribed nodule. Medullary is 2% of all breast cancers. Mucinous, papillary and tubular ductal carcinoma are also classified as IDC and can present with a circumscribed nodule.ILC13% of all breast cancers. Invasive lobular carcinoma 0-24% microcalcs. 30% multicentric, 10% bilateral. poor prognosis. 69% are palpable. Often occult in dense breasts. Architectural distortion is most common mammographic finding.IBCInflammatory breast cancer. Peau d orange appearance. Erythema without pain. Can be any histologic type although IDC is most common.Most common bilateral breast tumor which is also the least likely to present with a round mass?Lobular carcinoma in situ.Discussion-LCIS 10% have microcalcs. Arises in epithelium of mammary lobules. Not precancerous but marker of 10 fold increased risk of subsequent invasive cancer in either breast. Most common bilateral tumor 30-67% (statdx).Non-palpable and radiographically occult.What is incorrect regarding DCIS?Make up 20-30% of breast cancers- False30% become invasive after 10 years- True10% are palpable- TrueDiscussion-DCIS Approximately 10% of all breast cancers (Dahnert 567).72% have microcalcs. High nuclear grade (comedo type), necrotic, 90% calcified. Low nuclear grade (noncomedo type), non-necrotic, 50% calcified. Psammoma like bodies. Fine or coarse granular calcs. “arrowhead calcs” (Nick likes this one). 30% become invasive after 10 years. 10% are palpable.Invasive lobular carcinomaMost commonly radiographic finding is architectural distortion.Discussion-ILC13% of all breast cancers. Invasive lobular carcinoma 0-24% microcalcs. 30% multicentric, 10% bilateral. poor prognosis. 69% are palpable. Often occult in dense breasts. Architectural distortion is most common mammographic finding.Most common bilateral breast cancerLobulular carcinoma in situDiscussion-LCIS 10% have microcalcs. Arises in epithelium of mammary lobules. Not precancerous but marker of 10 fold increased risk of subsequent invasive cancer in either breast. Most common bilateral tumor 30-67% (statdx).Non-palpable and radiographically occult.19. Breast MRIDCIS shows rapid enhancement and washout- TrueKinetics are more important than morphology- FalseSclerosing adenosis is a benign cause of enhancement- TrueEnhancing septa in an oval lesion is a fibroadenoma- FalseDiscussion-DCIS can show slow initial enhancement with no washout (40-55%) which is typically a benign pattern or can show rapid enhancement with washout (30-50%) which is a malignant pattern. This is why MRI is not good for DCIS.The morphology always trumps the kinetics.Fibroadenomas can be with or without septations.The septations have mild contrast enhancement.20. Malignant lymph nodeBiopsy if rapid enhancement with washout- TrueMRI can distinguish between benign and malignant- FalseIf seen on the CC view, then it is level 2- TrueDiscussion-Need biopsy to distinguish between benign and malignant.21. Breast CA stagingPET is more sensitive than lymphoscintigraphy for staging the axilla- TruePET is more sensitive for IDC than lobular carcinoma- TruePET is more specific than lymphoscintigraphy- True22. Ultrasound shows fluid/fluid level in a breast massGalactocele- TrueLymphoceleComplex cystHamartomaDiscussion-Hamartoma- solid, heterogeneous, mixed or hyperechoic on US.23. Digital mammo compared to screen film.Digital has increased contrast- TrueDigital has decreased spatial resolution- TrueDigital has increased spatial resolution- FalseScreen film has lower contrast- TrueDiscussion-Digital has higher contrast and lower spatial resolution when compared to analog.24. Mammo must be overseen by a qualified radiologistMQSA act of 199225. GynecomastiaFlame shaped and retro-areolar in late stage gynecomastia- TrueAssociated with starvation, cimetidine, renal failure, testicular failure- TrueUS shows echogenic fatty material in retroareolar region- FalseMarijuana- TrueDiscussion-Early nodular- less than 1 year in duration, well demarcated subareolar mass.Late dendritic phase after 1 year, flame shaped, central subareolar mass.Diffuse glandular secondary to exogenous estrogen. Dense parenchyma, enlarged breast.Ultrasound shows hypoechoic material in retroareolar region.26. BIRADS lexicon, which is false?Heterogeneously dense breast tissue is >75% glandularDiscussion-<25% Almost entirely fat25-50% Scattered fibroglandular densities50-75% Heterogeneously dense>75% Extremely dense 27. Architectural distortion may be the first sign of malignancy- TrueBreast cancer risk factors, which is greatestLCIS- TrueAtypical ductal hyperplasiaAtypical lobular hyperplasiaInflammatory breast cancerCore biopsy is better than punch biopsy of the skin- FalseDiscussion-Want to do punch biopsy to look for malignant cells in skin lymphatics.Nipple inversion is not due to CongenitalAging- TrueInflammationAdenomaDiscussion-Aging should not cause retraction, this would be worrisome.Adenoma is a rare benign tumor of the breast that can cause nipple retraction.Phylloides tumorIncreased cellularity when compared to fibroadenoma- TruePartial resection- False5-10% of fibroadenoma convert to phylloides tumors- FalseDiscussion-Phylloides tumors have a more cellular stroma.Treatment is complete resection0.01- 0.3 percent of fibroadenomas can convert to malignancy (sarcoma)Chemotherapy can obscure cancer and change it’s kinetic curve from malignant to benign?- TrueDiscussion-This is how some institutions monitor the response of breast cancer to neo-adjuvant chemotherapy.Dense breast plus palpable lump and negative mammo, what is next step?UltrasoundCombined estrogen and progesterone hormone replacement.Decreases mammo sensitivity- TrueBirads 4Biopsy- TrueSpiculated lesion in the upper outer quadrantInvasive ductal carcinomaMRI appearance of fibroadenomaHyperintense on T1- FalseVery strong enhancement with type 1 or 2 kinetics- TrueAre taller than wide- FalseDiscussion-Iso or hypointense on T1Enhancing mass with minimally enhancing septationsType 1 kinetics- linear increase, <10% malignantType 2 kinetics- Plateau, approx 20% malignantType 3 kinetics- Up then down, >30% malignantFibroadenomas are wider than tall3rd most common breast lesion (cancer, fibrocystic, fibroadenoma).Most common benign solid tumor in young women.Multiple filling defects in the terminal duct lobular unitPapilloma- FalsePapillomatosis- TrueDCIS- FalseWell circumscribed soft tissue density in a 50 year old woman that turns out to be breast cancer is most likely to be?Medullary breast cancer- trueDiscussion-Medullary is a special type of IDC and is well circumscribed nodule.Ring enhancing breast lesion on MRI, which is true?Complicated cyst which is high on T2- TrueFibroadenoma- FalseMammographic appearance of invasive ductal carcinoma?Occult- FalseArchitectural distortion- FalseMalignant calcifications and spiculated mass- TrueDiscussion-Occult is typical of lobular carcinoma in situArchitectural distortion is typical of invasive lobularRegarding axillary lymph nodes?Mammo can’t tell benign from malignant- TrueIf MRI shows rapid washout, needs to be biopsied- TrueUS of malignant nodes shows lobulation of the edge- TrueDiscussion-One study showed that in women with malignant axillary lymph nodes and normal mammograms and ultrasounds, 83% showed suspicious breast lesions on MRI.43. A 1.4cm spiculated mass discovered on breast MRI w/ famhx of breast CA. Brought for biopsy and mass is not there. What next?Don’t do anythingReinject salineLook for cardiac enhancement. If not, reinject.- CorrectCorrect regarding breast cysts?Arises from terminal ductule within a lobule- FalseArise from epithelium- TrueTrue of Radial scars?Can have calcification and spiculation- TrueAssociated with sclerosing adenitis- TrueCan have calcifications and necrosis- FalseHas central malignancy (malignant nidus)- FalseAssociated with lactational adenoma- FalseDiscussion-Microcalcifications are possible but rare.No necrosisTrue regarding intracapsular rupture of implant?Rupture will rarely result after compression of 4 psi- TrueLess common than extracapsular rupture- FalseSilicon visualized external to capsule- FalseWater droplets seen in implant- TrueDiscussion-Mammograms are approximately 3-4 psi of compression. This rarely ruptures an implant but could if it is weakened. Most sources I’ve found say 10-15 psi required to rupture implant.Intracapsular- 80% of all ruptures, broken implant casing with silicon contained by fibrous capsule, linguini sign on MRI. Water droplets.47. Lesion should be within what distance on MLO and CC projections?1.0 cm- Correct0.5 cm1.5 cm2.0 cmWhich is true for breast conservation in breast CA?Can utilize neoadjuvant therapy to achieve- TrueRecommended to perform during first trimester- FalseMulticentric disease is not a contraindication- FalseMultifocal disease is not a contraindication- TrueIs contraindicated if there are palpable lymph nodes- FalseDiscussion-Can’t irradiate during pregnancy.Multicentric (multiple tumors in separate quadrants) is an absolute contraindication.Multifocal (multiple tumors in same quadrant) is a relative contraindication.49. A stereotactic breast biopsy performed for calcifications, but no calcs are seen by the pathologist on the tissue samples. What is the likely reason? The calcification is on the skinCalcium oxalate crystals- TrueNeed to advance needleLikely represent artifactsDiscussion-Calcium oxalate is radio-opaque and is seen on mammograms and during biopsy but they don’t stain so the pathologist can miss them if no polarized view is performed.50. Microcalcifications are biopsied, but not seen by pathologist on tissue specimen or in remaining breast tissue. What is the next step?Polarized view- CorrectReview with cytologistRe-biopsyNothingAdvance needleDiscussion-Calcium oxalate is radio-opaque and is seen on mammograms and during biopsy but they don’t stain so the pathologist can miss them if no polarized view is performed.Young female with erythematous, tender breastsGive antibiotics, if condition improves, no further follow up needed- TrueDo MRI- FalseBiopsy- FalseER/PR statusRecurrent and metastatic breast lesions have the same ER/PR status- FalseER/PR positive cancers are less aggressive than ER/PR negative- TrueRegarding BRCA mutations, which is false?Most diagnosed breast cancers from screening mammography are BRCA1 or BRCA2 positive- FalseDiscussion-BRCA mutations only in 5-10% of breast cancers. BRCA 2 is implicated in male breast CA.Which is false regarding the “halo” sign in breast imaging?Only seen with benign lesions- FalseDiscussion-Thick halo can be seen in breast CA.Enhancement on breast MR in post-operative bed 12 months out from surgery.RecurrenceDiscussion-Post operative enhancement is normal up until 6 months.Lobular round calcifications are associated with which of the following pathologic processes?LCISInfiltrative lobularDCISSclerosing adenosis- TrueDiscussion-Sclerosing adenosis- Lobular calcifications arise within spherical cavitySharply outlined, homogeneous, little variation in size.1.5- 2 x increased risk of malignancy.Which category is given to a breast lesion which is a known malignancy?BIRADS 658. Spiculated mass with pleomorphic calcifications in mammography?BIRADS 5Most common complication of breast surgery?Lymphedema- TrueSeromaBleedingInfectionBreast cancer with necrosis and calcification?Comedocarcinoma- TrueDiscussion-High nuclear grade (comedo type), necrotic, 90% calcified.Which benign condition has a 5 x increased risk of malignancy?Sclerosing adenosis- FalseAtypical ductal hyperplasia- TrueDiscussion-Sclerosing adenosis 1.5 - 2 x increased risk.Atypical ductal hyperplasia is the same as low grade DCIS.62. Incidence of Breast CA changes over time, which is true? Stage 1 increasing and Stage 2 decreasingStage 2 increasing and Stage 1 decreasing- TrueBoth increasingStable63. A core needle breast biopsy shows atypical ductal hyperplasia. What is the most appropriate treatment?Surgical excision- CorrectNothingSix-month follow-upUltrasoundStereotactic breast biopsy shows radial scar. What is the appropriate treatment?Surgical excision- CorrectNothingSix-month follow-upUltrasoundDiscussion-Dahnert says surgical excision is needed for definitive diagnosis.Architectural distortion seen on one view, what is the next step?Return to annual screening mammograms6 month follow upUltrasoundBring them back for XCCL and True lateral views- TrueBreast cysts arise from?Epithelium- TrueLobulesDuctulesApocrine cellsDiscussion-They are epithelial cells with apocrine metaplasiaRegarding CAD, which is true?Increases false positivesWhich portion of the breast is not well visualized on MLO view?Deep medial Cystic disease of the breastMost common in women <30- FalseEnhances homogeneously on T1 + c- FalseThick peripheral enhancement suggests simple cyst- FalseCysts originate from terminal lobule- TrueDiscussion-Most common in 35-55Patchy enhancementThick peripheral enhancement suggests complex cystCysts originate from epithelium in terminal ductule/ lobuleGastrointestinal Section1. Splenic trauma Grade: 2.5 cm laceration; and intraperitoneal fluid around the spleen with HU of 25. III- TrueIIIIVDiscussion-I- laceration <1 cm, subcap hematoma <25%II- laceration 1-3 cm, without involvement of vessels, subcap hematoma 25-50%III- laceration >3 cm, involves the vessels, subcap hematoma >50%IV- laceration involving segmental and hilar vessels with devascularization >25%V- shattered spleen.2. Feature of pancreatic microcystic cystadenoma that differentiates it from macrocystic cystadenoma?Solid appearing lesion pancreatic head- TruePain- TruePeripheral calcifications- FalseOccurring in the tail- FalseDiscussion-Microcystic/serous normally occurs in head of pancMacrocystic/mucinous normally in body or tail.3. Cystic lesion in panc head with lobulated borders most likelyPseudo cyst- FalseSerous cystadenoma- MaybeIPMN- MaybeDiscussion- Serous cystadenoma also commonly in head and has lobulated borders.IPMN usually in uncinate or head and have lobulated borders.4. Sigmoid volvulusHigh rate of recurrence- TrueIncreased risk with malrotation and long mesentery- FalseBE is contraindicated- FalseSeen in peds population- FalseDiscussion- Recurrence rate is 40-50%Cecal volvulus has increased risk with malrotation and long mesentery.Can do BE if low pressure.Older patients with chronic constipation.5. Best for confirmation of choledochal cystHIDA (may have said “DISIDA”)- FalseMRI- TrueCT with contrast- FalseUltrasound- FalseDiscussion-MRCP is actually the best imaging tool but of the answer choices MRI is best.I- Solitary Fusiform dilation of common duct (80-90% of choledochal cysts).II- Extrahepatic supraduodenal diverticulum.III- CholedochoceleIV- Intra and extra hepaticV- Caroli’s (only intrahepatic)6. Cystic mass pancreatic head with lobular contours and no wall enhancement Pseudocyst- FalseIPMN- MaybeSerous cystadenoma- MaybeMacrocystic cystadenomas are predominantly in women <35- FalseDiscussion-Pseudocysts and IPMN have enhancing capsules/septations.Microcystic/ serous neoplasms are benign and in 60-70 years old (grandmother).75% in women.Macrocystic/ mucinous cystadenomas mean age 40-50 years old (mother).99% in women.Solid pseudopapillary epithelial neoplasms are most common in young women <35 (daughter).Malignant gastric ulcersFolds radiate to edge of crater- FalseMost common in proximal half of stomach- FalseContrast beyond crater margin- TrueMost common in antrum- FalseDiscussion- BenignRound or ovoid with symmetric edge.Sharply defined defect, Projects beyond lumen of stomach. Lesser curvature, posterior wall or antrum.Proximal half of stomach in older patients.Hampton’s line- 1 mm thin straight line at neck of ulcer in profile view which represents the thin rim of undermined gastric mucosaMalignantUneven irregular shape with asymmetric edge.Interruption of radiating folds.Does not project beyond lumen of stomach.CT may show nodal, peritoneal or liver mets.Located anywhere, fundal ulcers above level of cardia are usually malignant.Carman meniscus sign- Ulcer is convex toward lumen of stomach.8. 20 year old girl gets CT abd/pel after MVA normal except for simple fluid in the cul de sac seen on 3 consectutive images ( 5mm slices )Call study normal. Say fluid is likely physiologic- TrueRepeat the scan with rectal and oral contrast- FalseFollow up ultrasound- FalseDiscussion- Simple fluid- physiologic or ascites.9. True of malignant GISTMost common in the duodenum- FalseMets to liver before local nodes- TrueLarge tumors don’t show necrosis- FalseDedifferentiation of neural and muscle cells- TrueDiscussion-Most common in stomach- 47-60%.Small bowel- 30%.They commonly show necrosis.10. Likely to be the source of bleeding after negative EGD and colonoscopyAngiodysplasia- FalseSmall bowel neoplasm- TrueDiverticulitis- FalseVillous or Adenomatous polyps- FalseSmall bowel polyps- TrueDiscussion-Angiodysplasia commonly diagnosed on EGDDiverticulitis or polyps would be seen on a colonoscopy11. True of primary hemochromatosisHCC is most common cause death- TrueLow T2 signal in the pancreas- TrueDiscussion-No increased mortality unless develop cirrhosis.If cirrhosis, 20-30% die from HCC.12. Calcified liver capsule and septaeShistasoma hematobium- FalseShistasoma japonicum- True (best answer)Shistasoma mansomi- True?Clonorchis sinensis- FalseEchinococcus- FalseDiscussion-Shistasoma hematobium usually affects the urinary tract.Shistasoma japonicum affects SMV and can cause capsular calc. “Tortoise shell” calcifications of liver capsule.Shistasoma mansomi affects IMV and can cause capsular calcClonorchis sinensis affects and calcifies subcapsular liver and has filling defects in the bile ducts. Can also cause liver abscess.Echinococcus- cyst in liver, pulmonary cysts in lower lobes, air crescent sign, floating endocyst creates water lily sign.13. Normal cranial caudal dimensions of the spleen with hilum visible9 cm13 cm- True15 cm21 cmDiscussion- 12-14 is normal14. True of candidiasis Single column esophagram highly sensitive for diagnosis- FalseResponds quickly to antifungals- TruePrimarily involves lower 1/3 esophagus- FalseLesions shape- Diamond or circumferential- FalseDiscussion-Double contrast is 90% sensitiveInvolves upper half of esophagusSmall less than 1 cm plaques.Shaggy carpet or cobble stone appearance.Diamond shape is herpes or HIVCircumferential shape is chronic esophagitis or drug induced.15. Metastasis most likely to mimic hemangiomaIslet cell met- TrueDiscussion- Hemangioma and Islet cell mets are hyperechoic on ultrasound.Early arterial enhancement and delayed solid or ring enhancement.16. Least likely to cause a hyperechoic liver lesion on USLymphoma- TruePancreatic Discussion-Lymphoma is hypoechoic on USHypoechoic liver lesions-Pancreatic adeno, lung, cervical, head and neck CA.Hyperechoic liver lesions-GI, Pancreas, Carcinoid, Choriocarcinoma, Mucin producing mets.Gastric diverticulum most common locationPosterior fundus- TrueAnterior antrum- FalseLesser curvature- FalseGreater curvature- FalseDiscussion-Seventy-five percent of true gastric diverticula were located juxtacardiac in the posterior wall of the fundus of the stomach, 2 cm below the GE junction and 3 cm from the lesser curve.Klatskin classification Periductal infiltrating is most common- TrueDiscussion-Extrahepatic cholangiocarcinomas are most commonKlatskin tumors are intrahepatic/hilar/central and at the confluence of the R and L hepatic ducts.GI tract bleeding Blood in NG tube aspirate excludes lower GI bleed- FalseLigament of Treitz separates upper from lower GI tract- TrueMost common cause of bleeding is from hemorrhoids- FalseMost common cause of bleeding is from Diverticulosis- TrueDiscussion-Could have upper and lower GI bleedLig of Treitz separates duodenum from jejunum but with regards to GI bleeding, also separates upper from lower GI bleeds.Hemorrhoids are not most common cause of GI bleeding.Diverticulosis is most common cause of GI bleeding in those over 40 y.o.Giardia Prox small bowel- TrueDiscussion-One of the only infections that affects the proximal small bowel. Duodenum and jejunum.Thickened folds 18 yo male with acute injury and no previous med history has free fluid in the retrovesicular space. No solid organ damage on IV contrast CT abdomen/pelvis.Small bowel injury is nonspecific on CT so an exploratory laparotomy is required to evaluate.- TrueSmall bowel injury is better detected with IV and oral contrast- FalseCT has 95% sensitivity for bowel injury- False probably75% have free air with bowel injury- FalseDiscussion-CT has 88-92% sensitivity for bowel injuryOnly 15-32% have free air with bowel injury.75% have intramural hematoma.Gallstone in duodenum, air in gallbladderBouveret’s syndrome- TrueMirizzi’s syndrome- FalseDiscussion-Bouveret’s- Gall stone erodes into duodenum causing gastric outlet obstruction.Mirizzi’s- compression of common hepatic duct by large gallstone in cystic duct, gallbladder neck or cystic duct remnant, accompanied by chronic inflammation.Gallstone ileus- most commonly gallstone is at terminal ileum.Rigler’s triad- intestinal obstruction, pneumobilia, ectopic gallstoneBilateral Adrenal hemorrhage adult Antiphospholipid antibody syndrome- True> 3 weeks of anticoagulation- FalseDiscussion-Antiphospholipid syndrome is a rare cause of bilateral adrenal hemorrhage.Anticoagulation is most common cause of bilateral adrenal hemorrhage but is almost always in the first 3 weeks of treatment.Trauma can cause adrenal hemorrhage but usually unilateral.Celiac diseaseIncreased number of jejunal folds and decreased number of iliac folds- FalseMost common in Asians and Africans- FalseHalf have splenic atrophy- TrueDiscussion-Reversal of normal folds leading to decreased jejunal folds and increased ilieal folds. Brooke prefers the term ilealization of the jejunum over jejunization of the ileum.Most common in caucasiansWhat is not seen with mesenteric carcinoid?Halo of sparing around mesenteric vessels- CorrectKinking, separation and displacement of small bowel loopsNarrowing and occlusion of vesselsDiscussion-Halo of sparing around mesenteric vessels is seen with mesenteric panniculitis.Have kinking of small and medium vessels. Also can have venous occlusion.Downhill varicesSVC obstructionDiscussion- Uphill varices: ↑ portal venous pressure → upward venous flow via dilated esophageal collaterals to superior vena cava (SVC)Distal 1/3 or 1/2 of esophagusMore commonDownhill varices: Obstruction of SVC → downward venous flow via esophageal collaterals to portal vein and inferior vena cava (IVC)Upper or middle 1/3 of esophagusLess commonAIDS patient with stenosis and dilation of intrahepatic bile ductsCryptosporidiosis- CorrectPrimary sclerosing cholangitisPrimary biliary cirrhosisDiscussion-Cryptosporidiosis causes irregular mild dilation of intrehepatic bile ducts resembling sclerosing cholangitis and has predilection for AIDS.AIDS causes secondary sclerosing cholangitis, not primary.Primary biliary cirrhosis not associated with AIDS and does not have stenosis.Most common type of intrahepatic cholangiocarcinomaMass formingIntraductal Periductal- TrueDiscussion-()Extrahepatic cholangiocarcinomas are most commonKlatskin tumors are intrahepatic/hilar/central and at the confluence of the R and L hepatic ducts.Small left colonMeconium plug- TrueBloody stool- FalseMaternal diabetes- TrueSoy bean allergies- FalseDiscussion-Small left colon is also known as meconium plug syndrome or functional immaturity of the colon (statdx)Most common association is maternal diabetesWhich is not a hypovascular liver metastasisPancreasBreastCarcinoid- CorrectColonDiscussion-Hypovascular- Stomach, colon, pancreas (adeno), lung, breastHypervascular- Neuroendocrine (islet, carcinoid, pheo), RCC, thyroid, choriocarc, melanoma, sarcoma, ovarian.Gall bladder cancerM > F 2-3:1- FalseGallstones are not important- FalseMass replacing GB- TrueEasy to differentiate from cholecystitis with wall thickening pericholecystic fluid and abscess formation.- FalseDiscussion-F > M 3-4:1Gallstones in 74-92% of cases of GB cancerMass replacing GB in 40-65% of casesDifficult to differentiate from cholecystitisMost specific finding in bowel ischemiaWall thickeningMesenteric edemaAbsent wall enhancement- CorrectDiscussion-Wall thickening is the least specific finding in acute bowel ischemiaMesenteric edema is 79% specific.Absent wall enhancement is the most specific at 96%Mesenteric fluid is the most sensitive.Spoke-wheel appearanceOncocytomaIncreased echogenicity of portal triads surrounded by edemaHepatitisDiscussion-Starry sky patternInsulinomas, all true except?Associated with MEN Type 1- TrueHyperdense on arterial phase CT- TrueMets to liver and regional LN’s- TrueMore common in females- TrueMost common in 60-70 year olds- FalseDiscussion-MEN 1- hyperparathyroid, pituitary tumors, Islet cell tumors.MEN 2- hyperparathyroid, Pheochromocytoma, Medullary thyroid CA. F>M, 3:2Most common in 4th and 5th decade of life.Most susceptible area in the colon for hypotensive ischemia?Splenic flexure and rectosigmoid- CorrectCecumTransverse colonDiscussion-Watershed areas are most susceptible and include-Griffith’s point- splenic flexureWatershed of SMA and IMA.Sudek’s point- rectosigmoidWatershed of IMA and hypogastricIleocecal False of benign gastric ulcersRound or oval shape- TrueContrast projects outside of lumen- TrueDiscussion-BenignRound or ovoid with symmetric edge.Sharply defined defect, Projects beyond lumen of stomach. Lesser curvature, posterior wall or antrum.Proximal half of stomach in older patients.MalignantUneven irregular shape with asymmetric edge.Interruption of radiating folds.Does not project beyond lumen of stomach.CT may show nodal, peritoneal or liver mets.Located anywhere, fundal ulcers above level of cardia are usually malign.Carman meniscus signMultiple hypodense round partially calcified liver lesions?EchinococcusDiscussion-Well demarcated round cyst with round peripheral cysts within the cyst (daughter cysts). Wall and septations can enhance.Partially calcified wall and internal septations.High density appearance of liver is seen on CT in all of the following except?Amyloidosis- CorrectAmiodarone treatmentHemochromatosisHemosiderosisWilson’sGold treatmentGlycogen storage diseaseDiscussion-Amyloid has regions of decreased attenuation with decreased enhancement.Acute appendicitis on radiographs.Accentuation of properitoneal fatAppendicolith in 50%Less visible fat- TrueDiscussion-Loss of properitoneal fatAppendicolith in 7-15%Loss of visible fat planes30 year old with innumerable colon polyps and osseous lesion of mandible. Appropriate follow up is?BE every 6 monthsColonoscopy yearlyColon resection with rectal anastamosisColon resection with anal anastamosis- CorrectDiscussion-Gardner’s syndrome- Adenomatous polyps with 100% malignant transformation, soft tissue masses and osteomas.Lesions with hamartomas- Cowden’s, Peutz Jager, Juvenile polyposis.Lesions with adenomatous polyps- Familial polyposis, Gardner’s, Turcot’s.Regarding esophageal perforation, which is True?Most commonly iatrogenic- TrueInitial study of choice is barium swallow- FalsePleural effusion can be seen- TruePneumomediastinum can occur- TruePleural effusions more common on the left- TrueMallory Weiss tear happens near the GE junction- TrueDiscussion-Use water soluble contrast, not barium.Distal esophageal perforations usually have left pleural effusion.Mid esophageal perforations usually have right effusions.Mallory Weiss tear- non transmural esophageal laceration.Boerhaave tear- transmural esophageal laceration.Which is least likely to cause cystic liver mets?Breast- CorrectColonOvaryDiscussion-Cystic liver mets- Mucinous ovarian, colon, sarcoma, melanoma, lung (SCC), carcinoid.Sacculation along antimesenteric border of the jejunum?SprueSclerodermaCrohn’s- TrueDiscussion-Scleroderma has sacculation along mesenteric sideCrohn’s has sacculation on anti-mesenteric sidePancreatic lesion with central scar and calcification?Serous microcystic adenomaAll present with small bowel dilation except?Eosinophilic gastroenteritis- CorrectWhipple’s- CorrectSprueParalytic ileusSclerodermaDiscussion-Eosinophilic gastroenteritis involves entire bowel, thickening and distortion of folds without dilation.Whipple’s- Irregular fold thickening, micronodularity without dilation.Sprue, Paralytic ileus and Scleroderma have dilated normal folds.Encapsulated liver mass with no bile ducts?HCC- correctFibrolamellarFNHHepatic adenomaDiscussion-FNH has incomplete capsuleAdenoma has pseudocapsuleLiver mass with tendency to hemorrhage?AdenomaPolyps in Peutz- Jeger syndrome are what type?HamartomaDiscussion- Most frequent polyposis of small bowel. Hamartomas, mucosal membrane freckles, adenomas of bronchus and bladder, Increased incidence of breast, pancreatic, GI cancer. Treatment includes polyp removal.Bullseye lesion in stomach on UGI?Metastatic melanomaDiscussion-Could also be Adenocarc, Lymphoma or GIST tumor.Pancreatic neoplasm aspiration labs show glycogen?Serous cystadenoma/ microcystic adenoma.Discussion-Associated with VHL, no malignant potentialMacrocystic adenoma or mucinous cystadenoma of pancreas?Has malignant potential- CorrectMost common in elderlyMost common in head of pancreasVanishing pancreasShwachman-Diamond syndromeDiscussion-Rare congenital disorder characterized by exocrine pancreatic insufficiency, bone marrow dysfunction and short stature. 15mm broad based, smoothly marginated submucosal mass with 5mm central umbilication along greater curvature of stomach is most likely?Ectopic pancreasSubmucosal 2 cm gastric lesion in young lady with hematemesis is most likely?LeiomyomaDiscussion-2/3 of leiomyomas occur in stomach. 90% of submucosal masses.Most common benign tumor of SB.Hypervascular and can hemorrhage.Which is not associated with Beckwith-Wiedemann Syndrome?Gastric volvulus- CorrectMacroglossiaAdrenocortical carcinomaHepatoblastomaDiscussion-LGA baby, macrosomia, hemihyperplasia, macroglossia, ventral wall defects, increased risk for embryonal tumors (Wilms, hepatoblastoma, neuroblastoma, rhabdomyosarcoma, adrenocortical carcinoma)Most common location for duplication cyst in child?Ileum- CorrectEsophagusColonDuodenumDiscussion-33% Ileum20% Esophagus13% Colon5% DuodenumEsophagram with distal esophageal mucosal thickening and longitudinal nodularity?Reflux esophagitisOrganoaxial volvulus, which is true?Often occurs with large hiatal herniaDiscussion-Rotates along long axis- greater and lesser curvatures reversed.Due to diaphragmatic defect (hiatal hernia)Mesenteroaxial along short axisConed cecum with normal terminal ileum?AmebiasisMost common location in GI tract for lymphoma?StomachMost common cause of acute mesenteric ischemia?Embolic disease in SMAMost common cause of gastric outlet obstruction in adults?Peptic ulcer diseaseNon-mobile hyperechoic foci in gallbladder most likely represent?Cholesterol polyps- correctAdenomatous polypsDiscussion-74% of polyps are cholesterol polyps.7% are adenomatous polyps.Spigelian herniasDefect at semilunar lineRichter’s herniasAntimesenteric portion of bowel in herniaDiscussion-Usually femoral hernias in older womenPartial bowel obstructionFalse regarding sigmoid volvulus?Most common in young patients- FalseMore common than cecal volvulus- TrueDiscussion-Sigmoid in old patients and related to constipation.Sigmoid- 60-75%Cecal- 20-40%, younger 20-40 years old. Malrotation and long mesentery.Most likely to affect the left colon?AmebiasisTBSalmonellaShigella- CorrectDiscussion-Shigella causes wall thickening and distinct layer stratification in left colon.Multiple lesions coalesce into one large lesionHemangioendothelioma- TrueAdenomaFNHKaposi’sGIST pathology to suggest malignancy?No differentiation into muscle or neural tissueDifferentiation into muscle tissueDifferentiation into neural tissueDifferentiation into both muscle and neural tissue- CorrectPulsion diverticulumMost common in proximal 1/3 of esophagus- FalseLess common than traction diverticulum- FalseWide neck- TrueSkeletal muscle in wall- FalseDiscussion-Mid to distal 1/3 of esophagusChest Section1. Ratio of bronchiole to pulmonary artery distal to the lobar branchesEqual- ProbablyBronchiole larger- FalseArtery larger- FalseDiscussion-Ratio should be about 1.Bronchiole should be equal or less than artery in peripheral locations.2. Most true statement regarding Stanford A dissectionsAscendingAscending and descending- TrueDescending onlyDiscussion-Type A- Ascending w or w/o arch and descendingType B- Descending only.Debakey classification- BAD (both, ascending, descending).3. Rheumatoid nodules in the lungMay wax and wane- TrueMay have cavitation- TrueUpper lobe predominance- FalseUsually in older individuals- FalseNo pleural effusion is present- FalseDiscussion-Rheumatoid nodules are seen in less than 5% of RA patients.Wax and wane. Large variation in size 5mm to 7 cm.Most common in lower lobes.50% will cavitate.Subpleural lower lobe predominance.Most common in middle age 25-50 years.Pleural effusions are often present, more common in males. 4. Satellite nodules seen inSarcoid- TrueNocardia+ANCA diseaseWegener’sDiscussion-Satellite nodules- TB, Sarcoid, BAC, Aspergillosis, EG, Hypersens pneumonitis5. Most common location for traumatic aorta lacerationRoot- FalseDiaphram- FalseGreat vessels- FalseIsthmus- TrueDiscussion-Isthmus just distal to subclavian artery (88-95%) Dahnert 6595-9% ascending aorta immed above aortic valve.4.5% aortic arch w avulsion of brachiocephalic trunk.6. Predominantly lower lobe disesaseAsbestosis- TrueSarcoidosisSilicosisDiscussion-Upper lobe- Silicosis, Sarcoid, LCHLower lobe- IPF, AsbestosisSarcoid usually subpleural along lymphatic vessels. septal, peribronchial.7. Which is a cause of dilated pulmonary artery without stenosisOsler Weber Rendau- Hereditary Hemorrhagic Telangiectasia- FalsePolyarteritis nodosa- FalseTetrology of Fallot- FalseTakayasu- FalseBehcet’s- TrueWilliam’s syndrome- FalseDiscussion- Takayasu has stenosis of pulm art.Polyarteritis nodosa- doesn’t affect pulm artery.Osler Weber Rendau- Dahnert does say that 2% get large artery aneurysms.Tetrology of Fallot- has stenosis of pulm art.William’s syndrome- has pulm art stenosis.8. Cardiac calcificationsBest method uses Agaston scoring- TrueVolume averaging- FalseNot common in asymptomatic people- FalseDiscussion-Agaston- weighs each plaque by a cofactor derived from plaque atten coefficient.Larger field of view increased volume averaging and decreases sensitivity.34% of people with cardiac calcs are asymptomatic.9. Most common cause of SVC syndromeMalignancyDiscussion-Over 90% are caused by malignancy, of which, Bronchogenic carcinoma is 50%10. Which is true/false about false ventricular aneurysmsHigh risk of delayed rupture- TrueCauses of arrhythmias- TrueTypically occur at posterior lateral wall of left ventricle- TrueOccur anterolaterally- FalseDiscussion- If question was asking about true ventricular aneurysms, the risk of rupture part would be false. Both true and false ventricular aneurysms can cause arrhythmias.True aneurysms occur left anterior and apical.False aneurysms occur posterolateral wall of LV.11. Cardiac massesMyxoma more common in females - TrueMyxomas are heterogeneous on T1 and T2- TrueMyxomas are the second most common cardiac tumor- FalseMyxomas are malignant- FalseLipomas occur in the intraventricular septum- FalseMost common mets are from breast and lung- Maybe80% of atrial myxomas are in RA- FalseLipomas are the most common cardiac mass in children- FalseDiscussion-M:F 1:1.7Myxomas are most common cardiac tumor in adults.Myxomas are benign.Lipomas most common in interatrial septum.Most common metsMyocardium- Melanoma most common.Peri and epicardium- Breast and lung most common75-80% in LARhabdomyoma by far the most common in children.12. Arrhythmogenic right ventricular dysplasiaThinned right ventricular free wall- TrueSeptum is rarely involved- TrueMRI can be diagnostic- TrueIncreased signal on triple inversion images- FalseDiscussion-Cardiomyopathy characterized by fibrofatty degeneration of right ventricle leading to spectrum of arrhythmias including sudden cardiac death.Possible R ventricular aneurysm.Triple inversion images- Dark blood and fat.Fatty replacement in right ventricle is low signal.13. Findings of right heart strainBowed septum- TrueDiscussion-Leftward bowing of interventricular septum.Reflux contrast into IVC.RV:LV chamber size ratio greater than 1.14. Acute PE truePolo mint and tramtracking signs- TrueNarrowed vessel lumen- FalseObtuse margins to vessel- FalseDiscussion-Partial filling defect surrounded by contrast material "Railway track" sign and "polo mint" sign.Narrow vessel lumen- Chronic PEObtuse margins to vessel seen with chronic PE.15. Pulmonary sling passesLeft pulmonary artery arises from right main pulmonary artery behind trachea and anterior to esophagus- TruePosterior to esophagus- FalseDiscussion-On upper GI anterior indentation on esophagus.Invasive aspergillosis Nodules with ground glass halo in immunocompromised person Intralobular septal thickening Asbestosis- TrueRA- FalseSarcoid- FalseDiscussion- Sarcoid- interlobular septal thickening.Intralobular septal thickening-IPF/UIP, Asbestosis, Alveolar proteinosis, Hypersens pneumonitis.Interlobular septal thickening-Interstitial edema/hemorrhage, lymphangitic spread of CA, sarcoid, UIP, lymphoma, amyloidosis, RACentrilobular nodules BOOPDiscussion-Random nodules- Miliary TB, miliary fungal/ histoplasmosis, hematogenous metsCentrilobular- Infection, hypersens pneumonitis, resp bronchiolitis-ILD, asthma, COP/BOOP, hemorrhage, PJP, alveolar proteinosis, CMV, LAM, EG/ LCH.Perilymphatic- Sarcoid, Lymphangitic spread, silicosis, AmyloidLung cancer staging question of a 4 cm lesion surrounded by lung with no nodes and malignant effusion T2aN0M1aDiscussion-See question 31 for TNM staging discussion.20. Intralobar mass communicates with the brochial tree & has normal vascular supply and drainage but delayed clearance of fetal lung fluid (Answer- Type I CPAM)Usually presents with small cysts and a dominant cyst > 2cm- TrueNeonate with respiratory distress and cyanosis (in 1st week of life)- True10% dx in adolescents & adults (after 1st year of life)- TrueAll lobes equally affected (middle rare and 20% are multiple lobes)- True95% of congenital lung disease, 75% have polyhydramnios, 6% have oligohydramnios- FalseComposed of multiple small cysts- FalseDiscussion-Formerly called CCAM, now CPAM.25% of congenital lung lesions.They communicate with the tracheobronchial tree.CPAM I: cysts vary in size w/one over 2 cm. Most common type at 50-70%. Good post resectionCPAM II: numerous tiny cysts under 2 cm. Second most common at 40%. Poor prognosis because of associated anomalies.Renal agenesis, pulmonary sequestration, cardiac anomalies.CPAM III: looks solid because of very small microcysts 3-5 mm. High perinatal mortality. Only 10% of all CCAM’s. Poor prognosis because of pulmonary hypoplasia and hydrops.In 25%, there are associated anomolies such as cardiac malformation, pectus excavatum, renal agenesis, prune-belly, jejunal atresia, chromosome abnormalities, bronchopulmonary sequestrations.No lobar predilection.CT shows mucoid impaction of bronchus and surrounding hyperinflated lung. (Answer- Intralobar pulmonary sequestration).Nonfunctional lung segment- True80% are intralobar- TrueDiscussion- Intralobar clinically present with- Adults- Recurrent pneumonia/ consolidation in LL.Left LL 60%, Right LL 40%Usually not associated with other congenital anomalies.One large artery from aorta and numerous pulm veins.L (aorta) to L (Pulm vein) shunt causes high output failure.Nonfunctional- No communication with tracheobronchial treeExtralobar clinically present with-Neonate- Asymptomatic or respiratory compromise.An accessory lung with it’s own pleura.Left LL 90%, Right LL 10%Associated with diaphragmatic defects 20%, pulm hypoplasia 25%, bronchogenic cysts and cardiac anomalies.Multiple small systemic (branch off of aorta) arteries to systemic veins (IVC/ portal/ azygous).L (aorta) to R (IVC) shunt.Called BP foregut malformation if communicating with GI tract.A cardiac bronchus typically originates from ____?Right lateral wall of trachea- FalseMedial wall of superior segmental bronchus- FalseMedial wall of bronchus intermedius- TrueLateral wall of bronchus intermedius- FalseMedial wall of left main bronchus- FalseDiscussion:Cardiac bronchus- Off of medial wall of bronchus intermedius.M>FOnly true supernumerary anomalous bronchus.Pig bronchus-off lateral wall right tracheaTrue regarding Pneumatoceles: Most common cause in kids is Staph- TrueNatural course is typically to resolve- TrueAssociated with pneumothorax- TrueSeen with hydrocarbon inhalation- TrueDiscussion:Cystic air collections secondary to obstructive overinflationCan also be caused by strep, h. influenza, e. coli, klebsiella, and serratia.Regarding intrapulmonary bronchogenic cysts, which of the following is false.Usually asymptomatic- FalseThin walled cyst may demonstrate air fluid level- TrueMore common than mediastinal bronchogenic cysts- FalseAssociated with wall enhancement on CT- TrueSeptations are rare- TrueAre infected 75% of the time- TrueDiscussion-85% are mediastinal and are usually asymptomatic15% are pulmonary are usually symptomaticLined by bronchial/respiratory epithelium75% get infected.Regarding lung transplant, what is the minimal residual volume needed in one lung to maintain survival?400 mL500 mL800 mL- True1200 mLDiscussion-FEV1 must be >800 mL in residual lung to allow for lobectomy. Use pre-op FEV1 x % lung estimated post resection based on perfusion nucs scanMost common cause of pulmonary edema is?Acute myocardial infarct- TrueChronic renal failure- FalseElectrolyte abnormality- FalseCongenital heart disease- FalseMost common cause of pulmonary vascular congestion?Acute myocardial infarct- FalseChronic renal failure- FalseElectrolyte abnormality- FalseCongenital heart disease- TrueDiscussion-Most common congenital heart disease to cause PVC is L to R shunt28. Regarding pulmonary infarction, which is true?Most PE’s go on to infarcts- FalseWedge shaped density over time keeps same shape but decreases in size- FalseLeaves a linear scar at area of infarction- TrueAssociated with ASVD- FalseDiscussion- Linear scar is in lower lobes 70% of the timeFalse regarding tracheobronchial injuryL>R- False80% occur within 2.5 cm of the carina- TruePersistent air leak is the most common radiographic presentation- TrueCan present with Fallen lung sign- TrueIncreased mortality- TrueAir most commonly tracks to thoracic soft tissues- FalseDiscussion-Air most commonly tracks to the mediastinumMore common on right (80%)Fallen lung sign is when there is complete transection of a bronchus and the lung falls to a dependent peripheral location, it is not common.Traumatic diaphragmatic visceral herniation90% due to MVC- TrueMay be asymptomatic for months/ years- TrueMost common on left side- True Most diaphragmatic tears are more than 10 cm long- TrueDiscussion- Bergquist triad- rib fx, spine/ pelvic fx, diaphragmatic ruptureL:R 3:1Usually posterolateral left diaphragm medial to spleen in radial orientation.Can have strangulation of stomach > colon > SB > omentumBilateral in <4%Most become symptomatic in <3 years.Covered by omentum if on L and by liver if on R.Associated with pleural effusions, atelectasis and phrenic nerve palsy.30% mortality if unrecognized early.2.5 cm lung mass with ipsilateral enlarged lymph nodes but no distant metastasis. What stage?IIa- TrueIIbIIIaIIIbDiscussion-TNM Staging of Lung CancerInternational System for Staging of Lung CancerStageDescriptionStageDescriptionT1<3cm in diameter, surrounded by lung/visceral pleuraIAT1 tumor in the absence of nodal / extrathoracic metsT2≥3cm in diameter / invasion of visceral pleura / lobar atx / obstructive pneumonitis / @ least 2 cm from carinaIBT2 tumor in the absence of nodal / extrathoracic metsT3tumor of any size; less than 2 cm from carina / invasion of parietal pleura, chest wall, diaphragm, mediastinal pleura, pericardium; pleural effusion; satellite nodule in same lobeIIAT1 N1 M0T4invasion of heart, great vessels, trachea, esophagus, vertebral body, carina / malignant pleural effusionIIBT2 N1 M0 + T3 N0 M0N1peribronchial / ipsilateral hilar nodesIIIAT3 other than T3 N0 M0 / N2N2ipsilateral mediastinal nodesIIIBT4 / N3 diseaseN3contralateral hilar / mediastinal nodes; scalene / supraclavicular nodesIVmetastatic diseaseRegarding MRI and CT staging of lung cancer, which is false?MR is better than CT for pancoast tumor- TrueMR is better for chest wall invasion- TrueMR is better for vascular invasion- TrueMR is more sensitive than CT for mediastinal adenopathy- FalseCT has a lower specificity for detecting lung cancer than PET?TrueFalse of Large cell lung CATypically large bulky and central- FalseStrongly associated with smoking- TruePrognosis is poor- TrueLess than 5% of all lung cancers- TrueMetastasis are late/ rare- FalseDiscussion-Bronchogenic cancer- 95% of all lung cancer, related to smoking.Small cell- 20%, central, lymphadenopathy.Non small cell- 80%Adenocarcinoma- 30- 40% of all lung cancer, peripheralBronchoalveolar cell carcinoma.Squamous cell carcinoma- 30- 40% of all lung cancer, central, bronchial obstruction.Large cell- <5% of all lung cancer, Rapid growthlarge (50% larger than 6cm) and peripheral, early mets.Non-bronchogenic cancer- 5%CarcinoidAdenoid cystic carcinomaMucoepidermoid carcinomaAtypical carcinoidFemale more than male- FalseCommonly presents as recurrent pneumonia- True10-25% of carcinoids are atypical- TrueMost commonly endobronchial and central- FalseLess aggressive than typical carcinoid- FalseYounger patients- FalseDiscussion-No PET uptake90% of carcinoids are GI tract > lung > thymus > biliary > ovarian teratomasTypical carcinoid is peribronchial, centrally located and low grade. F > M (10:1)Not associated with smoking.Atypical carcinoid is more aggressive/ moderate grade and peripheral. M > F (3:1)Associated with smoking.Bronchioloalveolar carcinoma40% present as solitary pulmonary nodule- True60% present as multiple pulmonary nodules- FalseAssociated with smoking- True2nd most common cavitary lesion behind squamous cell carcinoma- TrueGrowth pattern is hilic- FalseTypically noninvasive- TrueUsually negative on PET- TrueDiscussion-Spiculated margin with sunburst appearanceTypically negative on PET- 55%Only 25-50% are associated with smoking but it still is.Most common lung cancer in non-smokers.Growth is lepidic, not hilic. Most common cause of mediastinal (alternate: Anterior mediastinal) lymphadenopathy?Hodgkin’s lymphoma- CorrectNon-Hodgkin’s lymphomaSarcoidMetastatic diseaseLeast likely to cause hilar enlargement/ adenopathy in AIDS patientPCP- CorrectCryptococcusKaposi’sLymphomaSarcoidLeast likely to present as a posterior mediastinal massDescending aortic aneurysmTB osteomyelitis/spondylitisForegut duplication cyst- CorrectProstate metsDiscussion-Although foregut duplication cysts can be in posterior mediastinum, they are rare.False of Pancoast tumorsCan be caused by any cell type (variation- are most commonly seen in more than one cell type)- TrueAssociated with squamous cell only- FalseOften symptomatic- TrueMost commonly present with the classic signs and symptoms (variation- most commonly present with Pancoast syndrome)- FalseCommonly present with Horner’s syndrome- False40-50% have an apical cap- TrueOccurs with less than 5% of malignancies- TrueDiscussion-Can be any cell type but squamous cell 50% and adeno 25%.90% have shoulder and arm painPancoast syndrome- shoulder/arm pain, muscle wasting of hand and Horner’sRare as only 25-50% have Horner’s syndrome.Horner’s syndrome- Involvement of sympathetic ganglion with ptosis and miosis of the ipsilateral eye. An apical cap is pleural thickening at lung apex.Pancoast tumor- 50% have an apical cap >5 mm. True regarding superior sulcus/ Pancoast tumors25% are superior sulcus- False40% present as apical cap- True5% of all lung cancers- FalseOnly squamous cell carcinoma- FalseMost commonly present with classic Pancoast syndrome- FalsePancoast tumors are by definition superior sulcus tumors.Apical cap 40-50%Pancoast tumors are 3% of all lung cancers.Not “only squamous cell”, can be squamous 50%, adenocarc 20%.Which is false of SVC syndromeCauses facial and/or arm swelling- TrueImproves with supine positioning- FalseDiscussion-Clinically present with- dyspnea, facial swelling, head fullness, cough, arm swelling, CP, dysphagia, orthopnea, distorted vision, hoarseness, stridor, H/A, nasal stuffiness, nausea, pleural effusions, venous distention neck and chest wall and MSCAggravated with supine positioning. 80% caused by malignant mediastinal tumors.Which does not present as a chest wall mass in a child (Alternate- Which is not a malignant chest wall mass in a child)?NeuroblastomaEwing’s sarcomaAskin tumorRhabdomyosarcomaMesenchymoma- Correct for both variations of questionDiscussion-Mesenchymoma is in infants in 1st year of life.Chest wall masses in children- Ewing’s is most common >Neuroblastoma Askin tumor causes malignant pleural effusions in young white females and is basically a Ewing’s sarcoma of the chest wall.MC cause of pleural metsLung- TrueBreastLymphomaOvarianStomachDiscussion- Lung 36% >Breast 25% > Lymphoma 10% > Ovarian 5% > Stomach 2%Round or nummular pneumoniaMost common cause is streptococcus pneumoniae- TrueMost common in posterior lower lobes- TrueChildren most commonly affected- TrueAssociated with asbestos exposure- TrueRapid change in size and shape- TrueDiscussion-Strep > H. influ > Myco TB > pneumococcus > klebsiellaSpreads via pores of KahnCan mimick bronchogenic CAVaricella zoster virus appearance of pneumonia on CXRPatchy diffuse airspace opacities similar to pulmonary edema- TrueBilateral perihilar disease- TrueLobar pneumonia- FalseMosaic attenuation- TrueDiscussion- Most commonly seen in lymphoma and immunocomp patientsWidespread nodules (30%) and small calcifications. Unilateral diaphragm paralysismortality approaches 9-50% in adultsClassic presentation of post-primary TBLymphadenopathyMiliary nodulesCavitation- TrueTuberculomaConsolidationDiscussion-Upper lobe caseous necrosis with cavitation is typical appearance.Miliary is rareTuberculoma is usually in primary Tb but can also be secondary.Which is not a typical radiographic finding for PCP?Pleural effusions- CorrectBilateral perihilar or diffuse interstitial thickeningGround glass consolidationPneumatocelesDiscussion-Pleural effusions in 18%Can have interstitial, ground glass or patchy appearance.Pneumatoceles 38%Which is not a cause of bronchiectasis?PCP- True50. False of Pulmonary Echinococcal diseaseMost commonly involves lower lobes- TrueWell defined cysts with fluid levels- TrueSingle mass at lung base CXR- TrueCT commonly shows many nodular lesions at bases- TrueCommonly demonstrates calcified cysts in upper lobes- FalseWater lily sign- TrueDiscussion-Lung lesions rarely calcify. Also, they are more common in lower lobes.A child with leukemia on methotrexate treatment demonstrates bilateral pulmonary nodules and a halo sign on chest CT. This is associated with?CandidiasisMucormycosisAspergillomaSemi-invasive aspergillosisAlergic bronchopulmonary aspergillosisInvasive Aspergillosis- CorrectStreptococcusCMVWhich is most commonly associated with chest wall invasion?Actinomycosis- TrueAspergillosis- FalseTuberculosis- FalseDiscussion-Blasto, crypto and TB can also invade chest wall.Idiopathic pulmonary fibrosis most commonly appears asPeripheral basilar fibrosis- TrueDiffuse fibrosis- FalsePerihilar disease with mediastinal calcifications- FalseSmall nodules- FalseDiscussion-30% present with peripheral basilar fibrosis and honeycombing.Only 15% present with small nodules.30 y/o female smoker with pneumothorax and irregular thick walled upper lobe cystsHistiocytosis X/ LCHDiscussion-Actually slight male predominance.Spares costophrenic anglesUL predominanceThick walled cysts15% present with pneumothoraxCan progress to fibrosis and honeycombing (8%)Characteristic stellate/starfish scars.32 y/o with pneumothorax, multiple thin walled lower lobe cysts, pleural effusion and fatty lesion (hyperechoic lesions) in the kidneyLAM, Tuberous sclerosis- TrueLCH, Tuberous sclerosis- FalseLCH, VHL- FalseLAM, VHL- FalseDiscussion-Exclusively in women unless also associated with TS.Diffuse involvementOften have pneumothoraxChylous pleural effusions and massive hemorrhageObstructive lung diseaseSmall air filled cysts.LymphadenopathyCan be associated with tuberous sclerosis (AML).Respiratory bronchiolitisDiffuse bibasilar small linear and nodular opacities- TrueScattered ground glass opacities- TrueDiscussion-Smoker’s bronciolitisCentrilobular nodulesPigmented macrophages in respiratory bronchioles.Random nodules- Miliary TB, invasive fungal (histoplasmosis), hematogenous metsCentrilobular- Infection, hypersens pneumonitis, resp bronchiolitis-ILD, asthma, COP/BOOP, hemorrhage, PJP, alveolar proteinosis, CMV, LAM, EG/ LCH.Perilymphatic- Sarcoid, Lymphangitic spread, silicosis, AmyloidSinusitis with cavitary lung nodulesWegner’sDiscussion- Triad of sinusitis, pulm symptoms and renal insufficiency.c-ANCAWhich does not cause satellite nodules?SarcoidWegener’s- TrueTBConglomerate masses- ?Bronchioloalveolar carcinomaDiscussion-Satellite nodules- TB, Sarcoid, BAC, Aspergillosis, EG, Hypersens pneumonitisWhich is a cause of solitary pulmonary nodules?SarcoidWegener’s- TrueTBConglomerate masses- ?Bronchioloalveolar carcinomaScleroderma in chestGround glass opacities- TruePleural thickening and effusions- FalsePeripheral basilar distribution- TrueDeath is often by aspiration pneumonia- TrueDiscussion-Dahnert says pleural reaction/ effusion is distinctly uncommon.Which is false regarding BOOP?Subpleural consolidation- TrueSubpleural lines- TruePerilymphatic nodules- False Irregular large nodules- False Discussion-BOOP has centrilobular nodulesUsually small 3-5 mm.A patient with RA and history of environmental exposure develops Caplan syndrome. The pulmonary manifestation is characterized by?Lung nodules- TruePleural effusions- TruePneumothorax- FalseLobar consolidation- FalseAdenopathy- FalseDiscussion-Caplan’s syndrome- Pneumoconiosis + rheumatoid arthritis in coal workers; concominant with systemic rheumatoid nodules; rapidly developing well-defined nodules 5-50mm; appear in crops predominately upper lobe and peripheryNodules are more common than effusions but both occur.Which disease does not affect the pulmonary arteriesTakayasu’s arteritisAmyloid- CorrectBehcet’sTBDiscussion-Amyloid Congo red stainMultiple nodules in tracheal wallHemoptysisPrimary 70%, Secondary is rareTakayasu’s arteritisLarge vessel arteritis most commonly affects aortaCan also affect pulmonary arteryStenosis, thrombosis and aneurysmsPulseless diseaseBehcet’sYoung 20-30 M>FAffects vessels of varying sizesTriad of oral ulcers, genital ulcers and ocular problemsCan affect aorta, svc and pulmonary arteryWhich disease causes randomly distributed pulmonary nodules?TB- TrueSarcoid- FalseHypersensitivity pneumonitis- FalseDiscussion-Random nodules- Miliary TB, Miliary fungal (histoplasmosis), hematogenous metsCentrilobular- Infection, hypersens pneumonitis, resp bronchiolitis-ILD, asthma, COP/BOOP, hemorrhage, PJP, alveolar proteinosis, CMV, LAM, EG/ LCH.Perilymphatic- Sarcoid, Lymphangitic spread, silicosis, AmyloidWhich does not cause centrilobular nodules?Hematogenous mets- True (Random)LCH/ Histiocytosis X- FalseBOOP- False Respiratory bronchiolitis- False LAM- FalseDiscussion-Random nodules- Miliary TB, Miliary fungal (histoplasmosis), hematogenous metsCentrilobular- Infection, hypersens pneumonitis, resp bronchiolitis-ILD, asthma, COP/BOOP, hemorrhage, PJP, alveolar proteinosis, CMV, LAM, EG/ LCH.Perilymphatic- Sarcoid, Lymphangitic spread, silicosis, AmyloidWhich causes centrilobular nodules?BOOP- TrueSarcoid- FalseSilicosis- FalseDiscussion-Random nodules- Miliary TB, histoplasmosis, silicosis, hematogenous metsCentrilobular- Infection, hypersens pneumonitis, resp bronchiolitis-ILD, asthma, BOOP, COP, hemorrhage, PJP, alveolar proteinosis, CMV, LAM, EG/ LCH.Perilymphatic- Sarcoid, Lymphangitic spread, silicosis, AmyloidCauses of satellite nodulesAspergillosis- TrueBronchioloalveolar carcinoma- TrueEosinophilic granulomatosis- TrueHypersensitivity pneumonitis- TrueSarcoid- TrueWegener’s- FalseDiscussion-Satellite nodules- TB, Sarcoid, BAC, Aspergillosis, EG, Hypersens pneumonitisWhich is not a cause of intralobular septal thickening (or Parenchymal bands)?Pulmonary hemorrhage- True (probably best answer)TBLupusMetsSarcoidDiscussion-Pulm hemorrhage causes interlobular septal thickening.Intralobular septal thickening-IPF/UIP, Asbestosis, Alveolar proteinosis, Hypersens pneumonitis.Interlobular septal thickening-Interstitial edema/hemorrhage, lymphangitic spread of CA, sarcoid, UIP, lymphoma, amyloidosis, RAAce inhibitor side effectCoughAmiodarone can causeCOP/BOOPCystic fibrosis is not associated withLVH- Not associatedAsplenia- Not associatedCirrhosisBronchiectasisSinonasal polypsPancreatic diseaseMeconium ileusInfertilityHemoptysisSpontaneous pneumothoraxDiscussion-Cystic fibrosis associated with all of the above except LVH and aspleniaThey get right heart failure because of Cor pulmonale/ pulm arterial HTN.All cause bronchiectasis except?Kartagener’sCystic FibrosisTBPCP- CorrectScleroderma, which is false?Peripheral ground glass opacs- TruePleural thickening and effusions- FalsePulm function abnormal in absence of radiographic changes- TrueMost common location for congenital lobar emphysemaLeft upper lobeCause of pneumatoceleStaph aureusThe cardiothoracic ratio is >55% in which of the following?Mitral regurgitation- TrueAortic stenosis- FalseDiscussion-Mild to moderate cardiomegaly (Ratio 0.45- 0.55)Mitral stenosis, hypertrophic or restrictive cardiomyopathy, aortic stenosis, acute MI.Moderate to severe cardiomyopathy (Ratio >0.55)Mitral regurgitation, aortic insufficiency, dilated or ischemiccardiomyopathy, pericardial effusion.Behcet’sIritis, oral and genital ulcers, epididymitisDIP is most often seen inSmokers- TrueWhat is not a moderate contrast reaction?Mild laryngeal edemaTachycardia with mild hypotensionNausea and vomiting- TrueAirway compromise and hypotension are severe contrast reactions.N/V is mild contrast reaction.Lung cancer staging- 4cm mass, subcarinal nodes, malignant effusion.T2a, N2, M1aDiscussion-See question 31 for TNM classificationCardiac stress test/ perfusion imagingIf patient unable to reach max HR, may result in a false negative- TrueGive dobutamine for LBBB- FalseLBBB and paced ventricular rhythm have a high rate of false positive- TrueSestamibi is better than thallium because of redistribution- FalseDiscussion-Give adenosine for LBBBCauses of false negative- On beta blockers, Symmetric 3 vessel disease, Inadequate stress.No redistribution with Sestamibi vs ThalliumWhat is the most severe abnormality caused by oligohydramnios?Pulmonary hypoplasiaWhich is related to smoking?AIPDIP- TrueIPFUIPWhich is not true regarding Behcet’s?Multisystem disease- TrueMost common in men > 30 years old- False30% have vascular involvement- TrueIs associated with epididymitis- TrueDiscussion-Young 20-30 M>FAffects vessels of varying sizesTriad of oral ulcers, genital ulcers and ocular problemsCan affect aorta, svc and pulmonary arteryThymomaF>M- FalsePushes trachea to right- False15% of people with thymomas have myasthenia gravis- False30-35% invasive- TrueUsually in patients over 40 years old- TrueDiscussion-M:F, 1:1Pushes trachea posterior (not sure)35% of those with a thymoma have Myasthenia Gravis.15% of those with Myasthenia Gravis have a thymoma.Most common in 5th and 6th decades.Extralobar sequestrationLeft to right shunt due to systemic drainage- TrueDiscussion-Intralobar clinically present with- Adults- Recurrent pneumonia/ consolidation in LL.Left LL 60%, Right LL 40%Usually not associated with other congenital anomalies.One large artery from aorta and numerous pulm veins.L (aorta) to L (Pulm vein) shunt causes high output failure.Nonfunctional- No communication with tracheobronchial treeExtralobar clinically present with-Neonate- AsymptomaticLeft LL 90%, Right LL 10%Associated with diaphragmatic defects 20%, pulm hypoplasia 25%, bronchogenic cysts and cardiac anomalies.Multiple small systemic (branch of aorta) arteries to systemic veins (IVC/ portal/ azygous).L (aorta) to R (IVC) shunt.Called BP foregut malformation if communicating with GI tract.87. Mournier-Kuhn Syndrome Bronchiectasis and tracheomegaly- TrueDiscussion-Alternative recall described patient with 4cm trachea and bronchiectasis with tracheal collapse on expiration and asked for the dx of Mournier- Kuhn.TracheobronchomegalyDiscovered in 3rd to 5th decade. Marked dilation of trachea <29mmSitus inversus, bronchiectasis and sinus disease?Kartagener’sWhich is a cause of hyperlucent lung?Poland syndrome- CorrectCCAMPulmonary contusionPulmonary infarctionSequestrationDiscussion-Hyperlucent lung differential-Poland syndrome (absent pec major), Mastectomy, Hilar mass, Foreign body aspiration, Swyer-James, Emphysema, Pulm artery hypoplasia, PE, Congenital lobar emphysema.Correct regarding TAPVR (total anomalous pulmonary venous return)?Infracardiac type presents with congestion and pulmonary edema- TrueVenous return to left atrium- FalseInfracardiac is most common- FalseInfracardiac presents with early CHF- True Discussion-Needs ASD to survive because pulmonary veins drain to R atriumSupracardiac (type 1) Pulmonary veins to vertical vein to L brachiocephalic vein to SVCMost common 52%Cardiac (type 2) Pulmonary veins to coronary sinus, 2nd most common 30%Presents with early CHFInfracardiac (type 3) Pulmonary veins course esophagus and enter IVC or portal veinLeast common at 12%Severe early CHF and death within a few daysMost common sign of thoracic aortic injuryPseudoaneurysm- CorrectIrregular contourKinkingApical capDiscussion-Kinking is coarctationApical cap with transection92. The most common cause of a cardiothoracic ratio is >55%?Ischemic Cardiomyopathy- TrueHypertrophic CardiomyopathyRestrictive CardiomyopathyHypertensive CardiomyopathyDiscussion-Mild to moderate cardiomegaly (Ratio 0.45- 0.55)Mitral stenosis, hypertrophic or restrictive cardiomyopathy, aortic stenosis, acute MI.Moderate to severe cardiomyopathy (Ratio >0.55)Mitral regurgitation, aortic insufficiency, dilated or ischemiccardiomyopathy, pericardial effusion.Pediatrics Section1. Glenn shunt involves communication of Subclavian artery to pulmonary artery - FalseRight pulmonary artery to SVC- TrueSubclavian artery to right atrium- FalsePulm artery to aorta- FalseIVC to pulmonary vein- FalseHepatic to portal veins- FalseDiscussion-Glenn shunt is R pulm art to SVCFontan procedure is R atrium to pulm trunkBlalock-Taussig Shunt is Subclav art to pulm art 2. True of pediatric pneumomediastinumCan mimic medial PTX- TrueIf kid under 3 need to worry about aspirated foreign body- TrueClinically significant in full term neonate- FalseDiscussion-Any obstruction can increase intra-alveolar pressure and cause air to track through interstitum into mediastinum. 3. Retinoblastoma Commonly involves the optic nerve- TrueFinding of outpouchings of the vitreous- TrueDiscussion-Trilateral retinoblastoma- Involves bilat retinae and pineal glandQuadrilateral- all of above and suprasellar tumor.4. Transient/ Toxic synovitis of the hipPatients most commonly less than 6 years old- MaybeUsually bilateral (alternate- bilateral in 50%)- FalseFemale > Male- FalseMost progress to AVN- FalseDiscussion-Peak age is 6- Dahnert, usually 5-10 years old. Stat DX says 4-7.Normal radiograph on imaging is common.Recent viral illness, mild fever in 25%. Unilateral. M:F, 2:1Most have full recovery within a few weeks.Legg calve perthesCoxa vara- False4-8 years old- TrueBilateral 5-10%- TrueCalled Chandler’s disease if in an adult- TrueBilateral 50%- FalseFemale > Male- FalseDiscussion-Idiopathic AVN of femoral heads.Coxa planaPeak age 4-8 years.5-10% are bilateral.M:F, 5:1Called Chandler disease if in adults.6. Which is false regarding Medulloblastomas?Arises from roof of 4th ventricle and causes hydrocephalus- TrueHypointense on T1, Heterogeneous enhancement- False50% are hyperdense on CT without contrast- TrueDemonstrate dropped mets- TrueDiscussion-Most common neoplasm of post fossa in childhood.In roof of 4th ventricle.97% enhance as it is usually a solid tumor.Although they are typically hypointense on T1, the enhancement is homogeneous in 97%.70% are slightly hyperdense without contrast.7. True regarding PDA (patent ductus arteriosus)Functional closure by 6 hours- FalseCan have left to right and bidirectional flow in cases of respiratory distress syndrome- TrueProstaglandins used to keep open- TrueCloses with high oxygen- TrueRemains open with low oxygen levels- TrueDiscussion-Closure by 18-24 hours.Can have R to L flow with pulmonary arterial hypertension which can be caused by respiratory distress.Prostaglandin E2 used to keep open.Indomethacin used to close.High oxygen stimulates production of bradykinins and closure of PDA.Low oxygen causes the opposite.8. True regarding CCAM/CPAMConnection with the trachea- FalseCaused by abnormal proliferation of bronchial tissue- TrueMore common in ??? area of lung- FalseMost common type is microcystic and appears solid- FalseDiscussion-Communicates with bronchial tree, not tracheaNo lobar predilection.Most common type is Type 19. Most common non-cardiac intra-thoracic congenital defectCongenital diaphragmatic hernia- TrueCCAM- FalseSequestration- FalseDiscussion-Answer choices are in order of Hernias on L 75-90%, on R 10%, bilateral 5%.10. ThymusDisplaces the trachea- FalseCan be mistaken for right upper lobe pneumonia- TruePeak weight of the thymus is at puberty- TrueCommonly seen on CXR after 5 years old- FalseDiscussion-Normally located enlarged thymus does not displace trachea or SVC.Only common to see on CXR < 2 years old.11. Pediatric neck lesions are most commonly Congenital/ inflammatory- TrueVascular- FalseMalignant- FalseLymphatic malformation- FalseDiscussion-80% are congenital or inflammatory.12. Complication stubbed toe in kidOsteo- TrueEpidermal inclusion cyst- FalseDiscussion-Epidermal inclusion cyst could only form from a puncture wound in which skin gets inside of the bone.13. True of holoprosencephalySylvian fissures displaced posteriorly by hypoplastic frontal lobes- FalseCorpus callosum replaced by fused anterior frontal lobes- TrueFused frontal lobes- TrueThalamic fusion- TrueDiscussion-Sylvian fissures displaced anteriorly.Fused frontal lobes.Failure of basal forebrain structures to cleave.Severity of malformation related to degree of anterior brain developmentAlobar (least differentiated)- Absent IHF/falx and CC with pancake-like mass of brainSemilobar- IHF/falx formed posteriorly; splenium CC present; fused caudate headsLobar (most differentiated)- IHF/falx extend anteriorly; genu CC aplastic/hypoplastic; minimal frontal lobe fusion14. Cystic hygromas with diffuse body lymphedemaTurner’s- TrueTrisomy 18Trisomy 13Trisomy 21Discussion-Turner’s- Webbed neck, Osteoporosis, Short 4th metacarpal, Madelung deformity- bayonette wrist.Trisomy 18- Edward’s- Cystic hygromas (20%), Rockerbottom foot, Strawberry shaped skull. Choroid plexus cysts.Trisomy 13- Patau Syndrome- Holoprosencephaly, Cardiac defects, Enlarged kidneys, Polydactyly.Trisomy 21- Down’s Syndrome- Can have cystic hygromas but not the edema.15. Ten week old with solid renal fossa mass. No appreciable normal renal tissue identified.Mesoblastic nephroma- TrueCongenital Wilms- FalseRenal cell carcinoma- FalseNephroblastomatosis- FalseNeuroblastoma- FalseDiscussion- Mesoblastic is most common renal mass in neonates (1-3 months).Wilm’s is age 3 mo- 11 years but most common around 3-4 years.Nephroblastomatosis- A Wilm’s tumor under 2 cm is called nephroblastomatosis. Multiple, bilat nephrogenic rests and Beckwith Wiedemann.Neuroblastoma- peak age is 2 years.16. Renal lymphoma on US in kidsSimilar appearance in kids and adults- TrueHyperechoic mass- FalseRenal enlargement- TrueDiscussion-Lymphoma is hypoechoic on ultrasound.17. Best reasoning regarding imaging in pediatric patient with renal stones and renal disease.If US neg then don’t need further imaging as likely to be normal- TrueGet thin section CT- False18. Most common type of ventricular septal defect (VSD).Supracristal- FalseMembranous/ Perimembranous- TrueMuscular- FalseDiscussion-Perimembranous- 80% (most common)Muscular/trabecular- 5-10%Supracristal- 5-7% (least common)Palatine and adenoid tonsillar hypertrophy Strep A- TrueMono- TrueAdenovirus- TrueGastroschisis appearanceParaumbilical defect- TrueMembrane covering- FalseUmbilical cord inserts on apex- FalseDiscussion-Gastroschisis- Congenital defect, to right of umbilicus, no covering.Omphalocele- Congenital defect, involves umbilicus, covered. Worse because it is associated with cardiac and neural tube defects.Which is not a common cause of leukocoria?Coloboma- CorrectAcute retinal detachment- Also correct in a different yearPHPV (persistent hyperplastic primary vitreous)RetinoblastomaSclerosing endophalmitis (toxocariasis)Discussion- Causes of leukocoriaCataract, retinoblastoma, toxocariasis, coats disease, chronic retinal detachment, retinal dysplasia, Norrie’s dz.Coloboma can be a cause of leukocoria but is not as common as the other ones.Thanatophoric dwarfs Curved femursDiscussion-Thanatophoric- Telephone shaped femurs, narrow chest, short ribs,platyspondyly, cloverleaf skull. Similar to achondroplasia but more severe, lethal.Achondroplasia- accordion shaped femurs, champagne pelvis, flat acetabular roof, decreased intrapedicular distance, congenital canal stenosis, small foramen magnum, craniocervical instability, posterior vert body scalloping. rhizomelic shortening. Normal life spanOsteogenesis imperfecta- accordion femurs.Aicardi syndromePartial or complete absence of corpus callosum.Discussion-Spasms/ seizuresDistinctive chorioretinal lacunae are pathognomonicFontan shuntNo answers recalledDiscussion-Glenn shunt is R pulm art to SVCFontan procedure is R atrium to pulm trunkBlalock-Taussig Shunt is Subclav art to pulm artMost common right aortic arch anomalyRight arch with aberrant left subclav art- TrueDouble aortic arch- FalseRight arch with mirror image branching- FalseDiscussion- Most common is r arch w aberrant left subclav art- 35-72% of r arch anomalies Also, r arch with aberrant left subclav is second most common type of complete vascular ring behind double aortic arch (55% of all vascular rings).2nd most common r arch anomaly is r arch with mirror image branching- 24-60%Regarding an intussusception in a peds patientValsalva increases the risk of perforation- FalseValsalva decreases the risk of perforation- TrueMust keep pressure below 120 mm H20- True3-5 attempts- FalseDiscussion-Maximum attempts 3Valsalva increases extraluminal pressure which decreases colonic transmural pressure gradient and therefore decreases the risk of perforation.What doesn’t cause leukocoriaColobomaRetinoblastomaPHPVRetrolental fibroplasiaDiscussion-All can cause it?Coloboma- developmental causeRetinoblastoma- most common cause 58%PHPV (persistant hyperplastic primary vitreous)- causes leukocoria with unilateral microphthalmia, 2nd most common cause.Retrolental fibroplasia- a cause in 3-5% of casesAll are chilhood hemolytic causes of cholecystitis except?Hereditary spherocytosisErythrocytosis fetalisBudd chiari- correctThallesemiaDiscussion-Hemolytic diseases overwhelm the liver and cause an increase in unconjugated (as well as conjugated) bilirubin. This predisposes to stone and sludge formation.False regarding meconium aspiration?Persistent fetal circulation- TrueDiffuse coarse patchy opacities- TrueLow lung volumes- FalseSimilar appearance to hyaline membrane disease/ RDS- FalseDiscussion-Hypoxemia in utero leads to gasping reflex and meconium aspiration. This causes pneumonitis and pulm arterial hypertension which leads to persistent fetal circulation (R-L shunt).Air trapping from meconium aspirate leads to increased lung volume.RDS has decreased lung volume.Regarding Chiari 1 malformation?Short clivus- TruePeg like tonsils with CSF surrounding the tonsils- FalseHeadache/syncope with sneezing or coughing- TrueSmall posterior fossa- TrueUpper cervical cord surrounded by copious CSF- FalseDiscussion-Peg like tonsils refers to tonsillar ectopia, not a Chiari 1 malformation.Chiari I- More than 3mm protrusion of cerebellum below foramen magnum.Short clivusHeadache/ syncope after sneezing or coughing.Small posterior fossa.Thanatophoric dysplasiaMost common skeletal dysplasia- FalseAutosomal recessive- FalseCurved appearance of femurs- TrueLong bones appear short and wide- TrueCloverleaf skull- TrueDiscussion-Lethal, Severe rhizomelia (micromelic dwarfism)Second most common bone dysplasia after osteogenesis imperfectaRespiratory distress, hypotonic, protuberant abdomen.Pathognomonic- narrow chest, short ribsTelephone receiver humerus and femurs.Kleeblattschadel/ Clover leaf skullMost common soft tissue tumor in childrenRhabdomyosarcoma- TrueDesmoidSubglottic hemangiomaDiscussion-Most common soft tissue sarcoma in childrenSoft tissue mass with variable enhancementBone destruction/ remodeling possible.40% in head and neck.Congenital arrhythmic right ventricular dysplasiaThinning of right ventricular free wall- TrueSeptum and left ventricle involved- TrueDiscussion-Apoptosis and fatty infiltration of myocardium.Most common in right ventricle but can extend into septum and left ventricle.Fibromatosis coliAssociated with birth trauma- TrueBest diagnosed with ultrasound- TrueCan have unilateral torticollis- TrueDiscussion-Birth trauma causes compartment syndrome with pressure necrosis and fibrosis of unilateral sternocleidomastoid muscle.Right > LeftUltrasound shows mass in SCM and is the best imaging modality.14-20% have torticollisCephalohematoma is located/ affects which layers?Between periosteum and skull- TrueDoes not cross sutures- TrueSubgaleal- FalseBetween skull and dura- FalseInterhemispheric- FalseDiscussion-Layers- from superficial to deepSkinEpicranial aponeurosisPeriosteumSkullDuraCephalohematoma is between Periosteum and skull and does not cross sutures.Subgaleal hematoma is between Epicranial aponeurosis and Periosteum and can cross sutures.Which of the following is not a cause of hyperinflation in a child?Pulmonary hemorrhage- CorrectAsthmaPneumoniaDehydrationDiscussion-Hyperinflation in a child-Bronchiectasis, bronchiolitis, bronchopulm dysplasiaUpper airway obstruction (vascular ring, laryngitis)Cystic FibrosisPneumoniaForeign body inhalationAsthmaDehydrationPulmonary hemorrhage can cause hyperinflation in a neonate.Down’s syndrome associated with all of the following except?14 ribs- FalseMultisegmented manubrium with 2-3 ossification centers- TrueFlared iliac wings (Mickey mouse ears)- TrueCraniosynostosis- FalseDiscussion-11 ribs with Down’s syndromeAlso associated with-AA subluxation, Endocardial cushion defects, flattening of acetabular roof.Severity of ventricular septal defect VSD is determined by?Size- CorrectLocationType of dwarfism associated with normal lifespan?AchondroplasiaDiscussion-Thanatophoric- Telephone shaped femurs, narrow chest, short ribs,platyspondyly, cloverleaf skull. Similar to achondroplasia but more severe, lethal.Achondroplasia- accordion shaped femurs, champagne pelvis, flat acetabular roof, decreased intrapedicular distance, congenital canal stenosis, small foramen magnum, craniocervical instability, posterior vert body scalloping. rhizomelic shortening. Normal life spanOsteogenesis imperfecta- accordion femurs.Hypoplastic left heart?Dilated ascending aorta- FalseAtresia of LV- TrueAtresia of aortic root- TrueSmall PA- FalseSmall RV- FalseDiscussion-Atresia of mitral valve, LV, aortic valve, ascending aorta.Dilation of LA and pulm arteryHypertrophy of RV.Premature closure of metopic suture with forward pointing skull?TrigonocephalyDiscussion-Scaphocephaly- Sagittal suture, Dolichocephaly (long skull), most commonBrachycephaly- Coronal or lambdoid suture, Flat head syndrome.Plagiocephaly- Unilateral coronal or lambdoid suture, lopsided skullTrigonocephaly- Metopic suture, forward pointing skullOxycephaly- Coronal &/or lambdoid and sagittal.Regarding Tuberous Sclerosis, which is true?Associated with chromosome 17- FalseCardiac myxomas- FalseIncreased CNS tumors- TrueAutosomal recessive- FalseDiscussion-Chromosome 17 is Neurofibromatosis Type 1Tuberous sclerosis is chromosome 9 or 16.Cardiac rhabdomyomasIncreased CNS hamartomas and subependymal giant cell astrocytomas.Autosomal dominant or spontaneous mutation.Tuberous scelrosis- Seizures, retardation, adenoma sebaceum, cortical or subependymal tubers, hamartomas, giant cell astrocytomas, renal AML’s, renal cysts, pulmonary LAM, Cardiac rhabdomyomas.Musculoskeletal section1. True regarding these tumorsPosterior elastofibroma is malignant- FalseAskin tumor is seen in kids and young adults- TrueDiscussion-Elastofibroma- Lower scapula, often bilateral and theoretically due to overuse. Fibrous so low signal on all sequences.Askin- Large extrapulmonary chest wall tumor. Type of Ewing’s. Made of neuroectoderm. Malignant. Poor survival.2. True/false of osteoid osteomaUsually smaller than 2 cm- TrueHot on flow/blood pool phase of bone scan- TrueSeen as lucency on xray- True When in joint can cause effusion and synovitis- TrueDiscussion-The nidus is a lucency but the osteoid osteoma is mostly scleroticHot on all phases of bone scan.3. True of Giant cell tumor in the spineMost common in the sacrum- TrueUsually in the posterior elements/ neural arch- FalseFluid/fluid level common- TrueDiscussion- Overall, most common in the knee (50%), Sacrum and Vertebrae (10%).In the axial skeleton giant cell tumors are most common in sacrum.Usually in vertebral body not posterior elements.4. Bennet lesion in the shoulder isCalcified mineralization posterior labrum (alternate recall says posterior ossification of the posterior band of the glenohumeral ligament)- TrueDescription of Perthes- FalseDescription of Bankart- FalseDiscussion-Bennet- Throwers exostosis, Calcified posterior band of the inferior glenohumeral ligament as a result of extra-articular posterior capsular avulsion injury.Perthes- variant of bankart. labrum torn from glenoid rim but labrum held in place by intact periosteum.Bankart- Complete anterior labral tear with detachment5. Described spinatus muscle abnormality like edema or atrophySuprascapular notch- TrueInvolves axillary nerve- FalseInvolves quadralateral space- FalseDiscussion-Suprascapular nerve entrapment in suprascap notch which causes edema/atrophy in supra and infraspinatus muscles.Spinoglenoid notch- volleyball players. only infraspinatus muscle affected.Axillary nerve- inferior paralabral cysts can ipinge as it goes through quadrilateral/quadrangular and cause edema/atrophy of teres minor and deltoid muscles.Parsonage-Turner- supraspin and infraspin edema from viral neuritis of suprascapular nerve.6. Osteomyelitis drainage from the boneCloacaDiscussion- Cloaca means sewerSequestrum- fragment of dead bone surrounded by granulation tissue.Involucrum- Layer of living periosteal new bone formed around dead bone.7. Not true of fibrous dysplasiaOften undergoes malignant sarcomatous degenerationDiscussion-0.5% undergoes malignant degeneration8. True of scaphoid fracturePresents with ulnar sided hand pain- FalseIf neg xray and high concern MRI is highly sensitive in the acute setting- FalseBone scan is highly sensitive in the acute setting- FalseDiscussion-Radial sided hand pain.MRI is only 36% sensitive for scaphoid fractures.Bone scan is highly sensitive but only after 48 hours (not acute)9. Tendons with normal surrounding fluid in the setting of effusionPopliteus- FalseExtensor carpi ulnaris- TrueFlexor hallicus longus- TrueLong head biceps- TrueDiscussion-Couldn’t find specific mention of popliteus tendon having fluid surrounding it so I’m assuming it is false. Also looked at MRI’s and it does not appear to have increased t2 signal in normal settings.10. Associated with ChondrocalcinosisAmyloidosis- TrueSickle cell- FalseDiscussion- Causes of chondrocalcinosisCPPD, OA, Trauma, Gout, Hyperparathyroid, Progressive systemic sclerosis.Rare causesAmyloid, oxalosis, ochronosis, DM, Hypothyroid, Wilson, Hemochrom.11. Fracture radial head and dislocated distal radioulnar jointEssex Lopresti- TrueMonteggiaGallezziDiscussion-Essex Lopresti- comminuted radial head fx with dislocation of distal radio/ulnar joint.Monteggia- Ulna fx with anterior dislocation of radiocapittelar jointGalleazzi- Distal radial fx with distal ulnar dislocation12. Osseous cysts at base of lunate with tear of TFCC, fluid crossing the distal radio-ulnar joint and degeneration of the ulna in the setting of positive ulnar varianceUlnar impaction syndrome- TrueUlnar impingement- FalseDiscussion-Ulnar impingement is seen with negative ulnar variance.13. Beurger’s diseaseStrong association with smoking- TrueDiscussion- Beurger’s disease is small to medium sized vasculitis causing stenosis and occlusion. Present with claudication which can progress to ischemic ulceration.Also known as thromboangiitis obliterans14. Patellar joint space narrowing with subtle calcific density in the menisciCPPD15. Nerve in the cubital tunnelUlnar- TrueRadial- FalseMedian- FalseTibial- False16. Cause of bilateral Achilles ruptureFloroquinolones- TrueHereditary xanthomas- TrueOveruse stress- TrueDiscussion-Other causes of achilles ruptureTransient hypercalcemia from hyperthyroidism17. True of Ewing’s SarcomaMost common sites are femur and pelvis- TrueT1 abnormality on MR- TrueMost common in Posterior elbow- FalseMost common in Humerus- FalseCan be seen in the flat bones- TrueMore common in <25 years old- TrueIs 10% of all bone tumors- TrueDiscussion-Femur 25%, Pelvis & Ileum 14%, Tibia 11%, Humerus 10%.Most common malignant bone tumor in kids, peak age 15 years.M:F 1:270% diaphysis or metadiaphysis of long bones.Has low T1 when compared to normal bone and muscle.4-10% of all bone tumors18. Paget’s disease of boneMost often polyostotic- TrueOsteoporosis circumscripta- TrueBone scan positive only in mixed and late phase- FalseEarly phase is productive- FalseDiscussion- Bone scan hot/positive in all phasesEarly phase is destructive/ lytic, then mixed, then late phase blasticMalignant transformation to osteosarcoma less than 1% 19. Findings of erosive osteoarthritis not seen in osteoarthritisGullwing erosions and acroosteolysis- TrueCentral erosions and ankylosis- TrueErosions and osteophytes- TrueDiscussion-Definitely get gullwing appearance due to central erosions.Can have acroosteolysis of the proximal portion of the distal phalanx.Definitely gets central erosions. Can have ankylosis according to stat DX.Erosions and osteophytes definitely occur.20. Not associated with ACL tearAvulsed anterior tibial tubercle- CorrectFracture posterior lateral tibial plateauOCD medial femoral condyle- CorrectDiscussion-Tibial tubercle not involved with ACL. Segond fracture- Lateral tibial avulsion from LCL is associated with ACL tears.ACL tear associated with OCD of lateral femoral condyle21. Patient with erlenmyer flask, vertebral compression fractures and diaphyseal AVN causing a “bone-in-bone” appearanceOsteopetrosis- FalseSickle cell disease- FalseGaucher’s- TrueDiscussion- Osteopetrosis can have Erlenmyer flask deformity and bone in bone appearance but not vertebral compression fractures.Thallasemia can also cause Erlenmyer flask deformity.Sickle cell disease- can get endplate H shaped comp fxs but not other choices.Gaucher’s disease is mucopolysaccaridosis with findings of Osteopenia, osteonecrosis, pathologic fxs, endosteal scalloping, erlenmeyer flask deformities and H shaped vertebral bodies.22. Sickle cell Lace-like reticulation- FalsePunched out lesions in the middle and distal phalanges- FalseErosive arthritis/ joint erosions- FalseAdults get urate crystal deposition in joints- TrueDactylitis- TrueDiscussion-Lace like reticulations are with sarcoid. Punched out lesions in mid and distal phalanges is sarcoid.Sickle cell does not cause erosive arthritis (9% can get septic arthritis).Typical findings with sickle cellbone infarcts, osteonecrosis, osteolysis. periosteal reaction, osteomyelitis.H shaped or fish mouth vertebral bodies.Can get secondary gout from sickle cell with urate crystal deposition.Dactylitis (sausage digit) differential-TB, Sickle cell, Psoriasis, Septic arthritis, Sarcoid, Leprosy, Reiter’s23. Jefferson fractureBurst fracture of C1 (Atlas)- TrueUnstable when involving the anterior longitudinal ligament- FalseDiscussion-Unstable when involves the transverse ligament.24. True regarding Osteopetrosis (Albers Schonberg)Osteosclerosis- TrueRugger jersey spine- FalseDiscussion-Osteopetrosis is same thing as Albers Schonberg.Hyperostosis, osteosclerosis.Bone in bone appearancePicture frame spine- Vertebral body cortex is sclerotic. Causes of picture frame spine- Pagets.Rugger jersey- Horizontal subjacent endplate sclerosis.Causes of Rugger jersey spine- hyperparathyroid, renal osteodystrophy.Sandwich vertebrae- endplates are densely sclerotic.Causes of sandwich vert- osteopetrosis.Bone scan will be super scan.25. Apperance of myositis ossificansXray shows peripheral calcification- TrueMR shows soft tissue mass with central calcification- FalseDiscussionShould look like bone- Peripheral cortex and eventually calcifies centrally.Should not have central without peripheral calc. Basically it begins calcifying peripherally and concentrically fills in. this is a don’t touch lesion- early soft tissue component may look aggressive and if you biopsy it it looks histologically exactly like an osteosarcoma. 20% will have no history of trauma. Follow-up, don’t amputate. 26. Which have fluid/fluid levels on MRI?ABC- TrueSBC- TrueTelengiectatic osteosarcoma- TrueGCT- TrueNOF- TrueEG- FalseNot a finding with lateral patellar knee dislocation Lateral patellar edema- CorrectMedial patellar edemaLateral femoral edemaDiscussion-Medial patellar facet and lateral femoral condyle edema28. Which is the least likely to be injured in Sinus Tarsi SyndromeLis franc ligament- CorrectPosterior tibial tendonAnterior tibiocalcaneal ligamentCalcaneofibular ligamentDiscussion-Involved structures-Cervical ligament, interosseous lig, fat, bone marrow, inferior extensor retinaculum. Also, definitely can involve the calcaneofibular ligament.Fat necrosisOnly due to trauma or surgery- FalseMRI shows rim enhancement and follows fat on all sequences- TrueHypointense signal within the mass- FalseDiscussion-Hyperintense signal on T1 and T2 as it follows fat signal.Osteoid osteomas in the spineAlmost always involves the neural arch- TrueLucent rim- FalseScoliosis in young adult- TrueDiscussion-50% neural arch (Dahnert)Sclerotic rim with central lucent nidus and occasionally central sclerotic dot.Scoliosis is concave toward the lesion.SarcoidosisMost common in the hands- TrueDiffuse lacelike appearance of the bones- MaybeCauses lytic and destructive lesions- TrueDiscussion-Small bones of the hands and feet (middle and distal phalanges)Reticulated “lacelike” trabecular pattern in metaphyseal end, not diffuse.Well defined cystlike lesions of varying size.Neuropathy-like destruction of terminal phalanges.Sclerosis of vertebral bodies.PsoriasisSubperiosteal resorption- FalseOsseous destruction of phalanges- TruePencil in cup- TrueDiscussion-Subperiosteal resorption is with hyperparathyroidismOsseous destruction and acroosteolysis occasionally occurs w psoriasisOther characteristics of psoriasisMCP, PIP, DIP jointsAsymmetric except the sacroiliitis is usually bilateral.Sausage digitBony ankylosisDestruction of 1st toe IP joint with periosteal reactionParavertebral floating osteophyte.Fournier gangreneTesticle and epididymis can be normal- TrueClostridium is the most common cause- FalseInfection spreads hematogenously- FalseFrequently see gas within the scrotum- TrueMost often treated conservatively- FalseFemales > Males- FalseDiscussion-Obliterative endarteritis with cutaneous and subQ necrosis and gangrene.2-3 cm per hour of fascial destruction.M>>>F40-60% diabetic, also more prevalent if immunosuppressed. Polymicrobial- e. Coli and Bacteroides most common.Surgical emergencyGas in scrotal wall and perineum. Scrotal wall thickening with normal testes.Most common torn ligament in the ankle?Anterior talofibularContains new living bone in an area of osteomyelitisInvolucrum- TrueCloacaBrodie’s abscessSequestrumWhich is not associated with secondary hyperparathyroidism?Soft tissue calcsRenal osteodystrophyBrown tumors- CorrectOsteopeniaDiscussion-Usually due to chronic renal failure. Low vitamin D leads to hypocalcemia and therefore hyperparathyroid.Radiographic findings of secondary are-Osteosclerosis/ Rugger jersey spine, OsteopeniaSoft tissue calcs,Causes of hip pain with hip prosthesis?InfectionBursitisLooseningHeterotopic bone formationAll of the above- CorrectDiscussion-Heterotopic ossification, Trochanteric bursitis, Prosthetic fracture, Cement fracture, Dislocation, Loosening (aseptic loosening)- 50% by 10 years, Infection (septic loosening)- 1-9%, S. epidermidis 31%, S. aureus 20%.Causes of increased bone density include?Osteopetrosis- TrueOsteomalacia- FalseMastocytosis- TrueHyperparathyroid- TrueDiscussion-Diffuse increased bone density-MyelofibrosisRenal osteodystrophyFluorosisNeoplastic- Primary, Mets, LymphomaMastocytosisPaget’sOsteopetrosisSecondary HyperparathyroidismDiffuse decreased bone density-OsteoporosisOsteomalaciaMyelomaPrimary HyperparathyroidismOsteogenesis imperfectaSudeck atrophy AKA Reflex sympathetic dystrophy or complex regional pain syndrome.Sclerosis- FalsePainless swelling- FalsePermeative articular appearance- TrueDiscussion-Caused by trauma 50%, Idiopathic 27%Osteopenia, painful swelling, edema, dystrophic changes of skin and nails, Raynaud’s.Best way to evaluate marrow abnormality on MRI?T1Leukemia bone involvementPeriostitis- TrueCystic lucencies- TrueDoesn’t have a propensity for flat bones- FalseDiscussion-It does have a propensity for long bones, flat bones and the spineDiscoid MeniscusMedial > Lateral- FalseBest way to diagnose is > 2 bowties on sagittal images- TrueOlder people present with pain in this region- FalseDiscussion-L>M 10:1Usually children and adolescents present with pain.Atlanto-axial subluxationDown syndrome- TrueJuvenile rheumatoid arthritis- TrueDiscussion-Differential for AA subluxationJRA, Ank spondylitis, Psoriatic, Lupus, Trauma, Retropharyngeal abscess,Rheumatoid, Down’s syndromeWhich is not matched properlyKeinboch- Scaphoid- IncorrectDiscussion-Keinboch- LunateFreiberg- 2nd or 3rd metatarsal headKohler- NavicularPanner- CapitellumSever- CalcaneusWhich is not a small round blue cell tumor?MedulloblastomaOsteoid osteoma- CorrectOsteomyelitisLymphomaBone islandLangerhans cell histiocytosis/ EGDiscussion-Small round blue cell tumors- LEMON mnemonic (too bad lemons aren’t blue or round, they are more of an elliptical shape)Leukemia, LymphomaEwing’s, EGMultiple myeloma, MedulloblastomaOsteomyelitisNeuroblastomaAnatomic relationships, which is false?Peroneus brevis is posterior to longus- FalseMembranosis is anterior to tendinosis- FalseHumphry’s ligament is anterior to Wrisberg’s- TrueSubscapularis anterior to rotator cuff muscles- TrueDiscussion-Brevis is anterior to longus.Membranosis is medial to tendinosis.Humphry’s anterior meniscal ligamentWrisberg posterior meniscal ligamentClay shoveler’s fractureStable- TrueSpinous process fracture lower cervical or upper thoracic- TrueSurgical treatment- FalseDiscussion-Treatment is rest.Which is not associated with osteopetrosis?Bone in bone appearanceErlenmeyer flask Osteonecrosis- CorrectClavicular dysplasiaSegond fracture, which is true?Calcified MCL- FalseValgus strain with internal rotation- FalseACL injury- TrueMedial tibial plateau- FalseInsufficiency fracture- FalseDiscussion-Avulsion fragment from lateral tibial plateau.Varus and internal rotation.Ultrasound Section1. Doppler ShiftSound frequency increases as object approaches the subject- TrueIs due to the size of the reflectors- False?Minimum shift at Cos 90 degrees- TrueReversed shift when blood is coming at the transducer- FalseIf moving toward transducer, doppler shift is negative- FalseDiscussion-No doppler shift at cosign of 90It’s not the size that matters, it’s the motion in the receptor.Max doppler shift at cosign of 0Reversed shift when blood is going away from the transducer.2. Quant of 1200 most likely to seeGestational sac- TrueYolk sacFetal poleDiscussion- 1/7/11 rule (Dahnert 1015)-at 1000 quant see gest sac around 4.5 weeksat 7200 quant see yolk sac around 5 weeksat 10800 quant see embryo and heart motion around 6 weeks3. Small testicular calcifications scattered in the testiclesMicrolithiasis- TrueDiscussion- F/U every 6 months as 40% have concurrent germ cell tumor and there is a 21.6 x increased risk of germ cell tumor.Etiology- defects in phagocytic activity of sertoli cells leaving degenerated intratubular debris behind.4. Appearance of the proliferative endometrium on USHyperechoic- TrueHypoechoic and thin- FalseHypoechoic and thick- FalseDiscussion-Menstrual phase- 1-5 days, thin echogenic line 1-4 mm.Proliferative 6-14 days, iso-hyperechoic and thickened 5-7 mm.Periovulatory 14, triple ring sign, echogenic, hypoechoic, echogenic up to 11 mm.Secretory 15-28 days, very echogenic and thickened, up to 16 mm.5. Follicular ovarian cysts (Alternate recall says Functional ovarian cysts)Nonfunctional- False>2.5 cm- TrueOvum that failed to release from the ovary- TrueDiscussion-Functional cysts- 3 typesCorpus luteum cystFailure of corpus luteum to regress after ovulation.Should regress by the end of 1st trimester or 13 weeks.Can have internal hemorrhage.Elaborates estrogen Theca lutein cystSecondary to overstimulation.Usually 2-3 cm.Follicular cysts Unruptured graffian follicle from anovulatory cycle.Elaborates estrogen.Usually 2.5- 10 cm.Dahnert (1054) typically resolve within 2 cycles.6. Thyroid US shows multiple hypodense nodules with echogenic foci/calcifications and single enlarged cervical nodeMedullary carcinoma- FalsePapillary carcinoma- TrueDiscussion-Papillary has calcs and typically mets to nodes.Medullary is hypoechoic with lymph node mets and not usually calcified.7. Neonatal US shows obliterated posterior fossa and concave frontal bonesChiari IChiari II- TrueChiari IIIChiari IVDiscussion- Chiari I- More than 3mm protrusion of cerebellum below foramen magnum.Short clivusHeadache/ syncope after sneezing or coughing.Small posterior fossa.Chiari II-Lumbar myelomeningocele with small posterior fossaTectal beakingColpocephalyLemon skull- concave frontal bonesBanana cerebellum- cerebellum smashedChiari III- Occipital myelomeningoceleSyringomyeliaTethered cordHydrocephalusChiari IV-Lack of cerebellar developmentNot compatible with life8. True of subclavian stealRetrograde flow in vertebral arteryDiscussion-Proximal subclav art stenosis or occlusion (proximal to origin of vert artery) causes collateral blood flow to arm via retrograde flow from vertebral artery.9. True of the internal as opposed to external carotid artery on USCan identify with temporal tap- FalseSuperficial to the external carotid- False10. Middle aged female hepatic US shows cystic mass with septations and mural noduleBiliary cystadenoma- FalseDiscussion- Cystadenoma- cystic mass with septations without noduleCystadenocarcinoma- cystic mass with septations and nodule/sUni or multilocular cystic tumor.11. Gallbladder carcinomaUsually presents as a mass filling the lumen- TrueCan usually be differentiated from acute cholecystitis with abscess on clinical grounds- FalseDiscussion-90% adenocarcinoma, 10% squamous75% have gallstones. Can be febrile as well.Gallbladder mass >1 cm.12. Cystic changes involving the mediastinum testisRete testesDiscussion- Dilated rete testes is called tubular ectasiaMiddle age to elderlyFrequently bilateral13. Corpus luteumSupply estrogen until uterus takes over- TrueReleases progesterone- TrueReleases bHCG- FalseDiscussion-Wikipedia says that corpus luteum releases progesterone and estrogen. Eventually the placenta assumes that responsibility.bHCG is released by placenta not corpus luteum.14. Periportal cuffingHepatitis- TruePortal venous gas- FalseDiscussion- Cuffing can be caused by Inflamm bowel dz, acute hepatitis, liver inflammation, cholangitis, acute cholecystitis, pancreatitis.15. Sonographic measurement of the biparietal diameter should include these structures in the image?Cavum septum pellucidum and lateral ventricles- FalseThalami and cavum septum pellucidum- TrueLateral ventricles and thalamus- FalseDiscussion-Dahnert (1018) says @ level of thalami and cavum septum pellucidum and sylvian fissures.16. Lower extremity venous USLack of respiratory variation suggests more proximal clotDiscussion-Proximal clot means decreased respiratory variation or phasic flow.Distal clot would show augmentation of flow.17. Postpartum chick with hypertension and other symptoms. Has T2 signal in the posterior parietal lobesPRES- TrueSinus thrombosis- FalseDiscussion-Sinus thrombosis- empty delta signPRES can be from pre-eclampsia or eclampsia, drug tox, uremic encephalopPRES has parieto-occipital T2/ flairTheca lutein cystsCommonly bilateral- TrueAssociated with PCOS- TrueUnilateral- FalseTendency to rupture- Discussion-Typically multiple and bilateral.Associated with ovarian hyperstimulation syndrome, PCOS and ovary stimulating drugs.Bilateral carotids demonstrate reversal of early diastolic flowAortic regurgitation- TrueInternal carotid artery stenosisExternal carotid artery stenosis(Acute?) DVT on US has all of the following exceptLuminal contraction- CorrectIncomplete compressibility of the veinLoss of respiratory/ cardiac cycleFixed echogenicity within lumen.Discussion-Luminal contraction is only seen with chronic DVT.The question probably asked about acute DVT.Maximum doppler angle60Cirrhosis on ultrasoundInhomogeneous with nodular contour- TrueInhomogeneous with smooth margins- FalseHyperechoic and heterogeneous- TrueHypoechoic- FalseGeneralized hypertrophy of the left lobe- TrueMultiple small splenic lesions in a 20 year old AA male with a normal sized spleen.Systemic candidiasis- TrueLymphoma- FalseSarcoidosis- FalseSickle cell- FalseTBEchinococcal cystsDiscussion-Dahnert says that candidiasis also can have splenomegaly but I would still choose that as the best answer.Lymphoma usually has splenomegalySarcoid usually has splenomegalySickle cell normally has splenic atrophyMost echogenic organs (most to least)Pancreas, Spleen, Liver, Adrenals, KidneysPapillary thyroid cancerMetastasizes to cervical lymph nodes- True80% of thyroid cancers- True (kind of)Least aggressive- TrueHematogenous spread to bone and brain- FalseDiscussion-Lymphangitic spread in 40% of adults and 90% of children.Papillary- 60-70% (80% is close enough)Follicular- 20%Anaplastic- 15%Medullary- 1-5%Papillary has the best prognosis.Hematogenous spread to lung (4%), bone is rare.Regarding chronic cirrhosis on ultrasound, all are true except?Left lobe hypertrophy- TrueLobular contours- TrueBecomes hypoechoic- FalseWhich may be the cause of an echogenic intracardiac focus on an obstetrical ultrasound?Down’s syndrome- TrueEdward’s syndrome- TrueTurner’s syndrome- FalseKlinefelter’s syndrome- FalseDiscussion-EIF (echogenic intracardiac focus) is a small bright spot on ultrasound thought to represent calcification in the myocardium. 3-5% of normal pregnancies. No impact on heart function. Associated with Trisomy 18 (Edward’s) and Trisomy 21 (Down’s) 4.8 x increased risk in radiographics article.Germinal matrix hemorrhage grade 3No answers recalledDiscussion-Grade 1- hemorrhage confined to germinal matrixGrade 2- Intraventricular hemorrhage without ventricular dilationGrade 3- Intraventricular hemorrhage with ventricular dilationGrade 4- Intraventricular rupture with hemorrhage in surrounding white matter.Beta HCG of 1800 what do you expect to see?Gestational sac- TrueYolk sacFetal poleDiscussion-1-7-11 ruleBelow 1000- nothing1000- gestational sac 4.5 weeks7000- yolk sac 5 weeks11000- fetal pole 6 weeksUltrasound characteristics of epididymitisEnlarged- TrueHypoechoic- TrueIncreased doppler flow- TrueWhich of the following is most characteristic of a malignant Thyroid lesion?Microcalcifications- TrueCystic componentHyperechoicThick HaloDiscussion-Complete halo is benign 90% of the time. Occasionally, papillary cancer can show an incomplete halo.Single cord artery/ vein is seen on prenatal ultrasound. What is the chance of a congenital anomaly?1020- Correct5090Discussion-21% chance of congenital anomaliesCV- Heart disease most common- VSDCNS- hydrocephalus, holoprosencephalyGU- hydronephrosisGI- Esophageal atresia, cystic hygroma, cleft lipMSK- Polydactyly, syndactyly, Ventral wall abdominal defectHighest complication rate is in which type of twin pregnancy?Mono/mono- correctDi/diDiscussion-Mono/mono 50% complication rateDi/di 10% complication rate

................
................

In order to avoid copyright disputes, this page is only a partial summary.

Google Online Preview   Download